Anda di halaman 1dari 77

A 55-year-old woman with a history of type II diabetes mellitus (DM), hypertens

ion, and obesity is seen in the clinic for general followup. Her current medicat
ions include metformin, lisinopril, aspirin, and citalopram. She reports well co
ntrolled fasting blood glucose values with AM fingersticks of approximately 100
mg/dL. Her blood pressure is controlled at 120/70 mm Hg. You order fasting lipid
s. Her laboratory studies show:
Which of the following is the most appropriate medication at this time?

A.
Garlic
B.
Gemfibrozil
C.
Niacin
D.
No medications are necessary at this time
E.
Simvastatin

The correct answer is E. Current guidelines (ATP III) consider DM to be a corona


ry heart disease (CHD) equivalent. In a patient with diabetes, therefore, the go
al LDL is <100 mg/dL. The most appropriate medication to achieve this goal in pa
tients with diabetes is an HMG-CoA reductase inhibitor (simvastatin), as evidenc
ed by the CARE trial and Heart Protection Study. Prescribing no medications (cho
ice D) is therefore incorrect.
Choice A is incorrect because there are presently no good data supporting the us
e of garlic in the treatment of dyslipidemia.
Although the patient does have elevated triglycerides, the first goal of therapy
should be treatment of the patients LDL cholesterol. Statins do have some effect
on lowering triglycerides. After normalization of LDL is achieved, one may cons
ider a second agent such as gemfibrozil (choice B) for treatment of hypertriglyc
eridemia. Obesity, excessive consumption of simple sugars and saturation fats, i
nactivity, alcohol consumption, and insulin resistance are commonly associated w
ith hypertriglyceridemia.
Niacin (choice C) can help to increase HDL levels but is not an optimal agent fo
r LDL reduction. At high doses it can increase HDL up to 30% but will decrease L

DL by only approximately 5-10%. Its use often is limited by side effects such as
flushing. It is also associated with insulin resistance and hyperglycemia, whic
h make it a suboptimal choice for this patient.
A 50-year-old African-American woman returns to the clinic for a follow-up app
ointment for diabetes treatment. At her last visit, she was screened for diabete
s because both of her parents and one of her two sisters have diabetes. After be
ing found to have repeatedly elevated fasting blood glucose levels, the patient
was given diabetic education and tried on a course of diet and exercise. While s
he has no specific complaints today and is pleased that she has lost 4 lb over t
he last 6 months, her fasting blood glucose is 130 mg/dL and a hemoglobin A1C le
vel is 7.7%. After a lengthy discussion with the patient, it is decided that she
should start therapy with metformin as well as continue her dietary controls an
d exercise regimen. Which of the following is the most common side effect for wh
ich she is at risk when starting this therapy?

A.
Anemia, megaloblastic
B.
Gastrointestinal distress
C.
Lactic acidosis
D.
Liver function abnormalities
E.
Weight gain

The correct answer is B. Metformin is one of the most common oral agents used to
treat noninsulin diabetes mellitus, necessitating an understanding of its sideeffect profile. The most common side effect is moderate to severe gastrointestin
al distress, which can cause patients to discontinue therapy. This can be avoide
d or minimized by starting at a low dose of metformin, taking the drug with meal
s, and titrating the dose slowly.
Metformin can reduce B12 absorption, though this effect is almost never clinical
ly significant. It is extremely rare for metformin to cause a megaloblastic anem
ia (choice A).
Lactic acidosis (choice C) is a rare but serious side effect of metformin, which

usually occurs only if predisposing factors are present. Common predisposing fa


ctors include renal insufficiency, heart failure, serious acute illness, or any
hypoxic state. It is recommended that metformin be held for any imaging study wi
th iodinated dye, to reduce the risk of having metformin present if contrast ind
uced nephropathy occurs.
Liver function abnormalities (choice D) are uncommon and do not need to be monit
ored for drug intolerance. Other oral agents, such as the thiazolidinediones (gli
tazones) require monitoring for hepatic complications.
Metformin will often result in a small weight loss, not a weight gain (choice E)
, unlike most of the other medications used to treat diabetes.
A 72-year-old woman is seen for followup of therapy for hyperlipidemia. She re
cently was started on pravastatin (40 mg/day) for this problem, and she does not
receive any other medications besides over-the-counter vitamin preparations. Sh
e is otherwise healthy, but has been complaining of fatigue. General physical ex
amination is normal. Laboratory testing yields the following results that are co
nfirmed on repeat testing:
Which of the following is the most appropriate management at this time?

A.
Add a second anti-lipid agent
B.
Measure anti-thyroperoxidase antibody levels
C.
Order a radioactive iodine uptake scan
D.
Stop pravastatin because of fatigue
E.
Treat the patient with thyroid hormone

The correct answer is E. This patient has what is commonly called subclinical hyp
othyroidism. This condition is defined by the combination of an elevated thyroid
stimulating hormone (TSH) level in combination with normal serum thyroxine level
s. Subclinical hypothyroidism is distinguished from overt hypothyroidism, in whi
ch the TSH is high and the thyroxin level is low. Many patients with subclinical

hypothyroidism are not truly asymptomatic. Rather, they may have a variety of s
ymptoms and signs that are seen also in overt hypothyroidism. These symptoms and
signs include mild hypercholesterolemia, fatigue, mild depression, and others.
When patients have test results indicating subclinical hypothyroidism and they h
ave symptoms or signs of overt hypothyroidism, they should be treated with thyro
id hormone.
Adding a second anti-lipid agent (choice A) is not appropriate because the patie
nt is not yet on a maximal dose of a single anti-lipid agent, and because subcli
nical hypothyroidism may be contributing to hyperlipidemia. Before further modif
ication of anti-lipid therapy, the clinician should determine if hyperlipidemia
resolves with thyroid hormone therapy.
Measuring anti-thyroperoxidase antibody levels (choice B) would be appropriate i
f the patient had no symptoms or signs of hypothyroidism. When patients have lab
oratory study results indicating subclinical hypothyroidism but no symptoms and
signs of hypothyroidism, anti-thyroperoxidase antibody levels should be measured
. The presence of these antibodies indicates a high likelihood that subclinical
hypothyroidism will progress to overt hypothyroidism (i.e., high TSH and low thy
roxin levels). If antibodies are present, the patient should be treated with thy
roid hormone to avoid developing overt hypothyroidism.
Ordering a radioactive iodine uptake scan (choice C) is inappropriate. Radioacti
ve iodine scans are used to evaluate patients who have hyperthyroidism or thyroi
d nodules. They are not used to diagnose hypothyroidism.
Stopping pravastatin (choice D) is not appropriate because the patient still req
uires anti-lipid therapy and is having no particular difficulty with pravastatin
. In fact, her liver function tests and creatine kinase levels are normal, exclu
ding two of the more common forms of drug toxicity from statins (i.e., hepatic d
ysfunction and myonecrosis, respectively). Fatigue likely is caused by subclinic
al hypothyroidism, rather than by pravastatin.
A 34-year-old woman complains of increasing urinary frequency. She also report
s several urinary tract infections over the past year. She has no prior medical
history and is not taking any medications. She denies weight loss. There is a ve
ry strong family history of diabetes. On examination she is noted to be obese. L
ung, heart and abdomen examinations are unremarkable. A fasting glucose is mildl
y elevated. The patient expresses concern about renal failure as her father and
brother undergo dialysis. Which of the following tests should be conducted to ev
aluate this patients risk for renal failure?

A.
Creatinine clearance
B.
Renal ultrasound
C.
Serum creatinine level
D.
Urinalysis

E.
Urine albumin level

The correct answer is E. Diabetic nephropathy may be functionally silent for 1015 years. Clinically detectable nephropathy is marked by microalbuminuria with 3
0-300 mg of albumin/24 hours. Microalbuminuria precedes nephropathy in patients
with diabetes mellitus.
The filtration rate in diabetics may be elevated at the time of onset of albumin
uria (choice A). Creatinine clearance is a reflection of the filtration and may
lag development of nephropathy.
A renal ultrasound (choice B) is a good test for evaluating hydronephrosis from
post obstructive renal failure but is not a good test for screening for diabetic
nephropathy. Changes consistent with diabetic nephropathy will require a renal
biopsy.
Azotemia begins a decade after the diagnosis of diabetes (choice C). Thus creati
nine level is not a reliable way to detect early diabetic nephropathy.
A urinalysis (choice D) with a urine dipstick may detect albumin. However, this
is a less sensitive test than the 24-hour urine collection study.
A 45-year-old man complains of severe cramps in his legs for the past 30 days.
He previously has been healthy with no complaints. He takes no medications, den
ies drug use, and rarely consumes alcohol. He has no chest pain or shortness of
breath. When specifically questioned, he reports that he has been feeling increa
singly fatigued. On examination, he appears lethargic. His vital signs are stabl
e. Lungs are clear. Cardiac examination is unremarkable. His muscle strength is
intact. Laboratory studies show a normal hematocrit, an elevated thyroid-stimula
ting hormone level, and normal T3 and T4 levels. Which of the following is the m
ost appropriate management?

A.
Methimazole therapy
B.
Observation
C.
Radioactive iodine therapy
D.
Thyroidectomy

E.
Thyroxine therapy

The correct answer is E. The patient is hypothyroid as shown by the elevated thy
roid-stimulating hormone (TSH). If his T3 and T4 were diminished also, this diag
nosis would have been certain. If he were asymptomatic and had normal T3 and T4
levels, then he would have subclinical hypothyroidism. He has normal T3 and T4 b
ut is symptomatic with leg cramps and lethargy. This should be treated with thyr
oxine, and the TSH level should be rechecked in 6 weeks.
Methimazole (choice A) would be used in treating hyperthyroidism. Agranulocytosi
s is a possible adverse side effect.
Observation (choice B) would be appropriate if the patient had subclinical hypot
hyroidism.
Radioactive iodine therapy (choice C) would be used for the treatment of Graves
disease.
Surgery would have no role in the initial management of hypothyroidism (choice D
).
A 48-year-old man comes to your office complaining of wrist pain and numbness
of the thumbs and index and middle fingers of both of his hands, which has been
worsening over the last 6 months. His only chronic medical condition is slight hy
pertension, for which he is being treated with hydrochlorothiazide. On further qu
estioning the patient states that he has noted progressive coarsening of his fac
ial features, as well as an increase in his shoe size, all of which has continuo
usly worsened. His blood pressure is 145/95 mm Hg, pulse is 95/min, respiration
rate is 15/min, and he is afebrile. He has a deep, hollow-sounding voice, and yo
u notice slight prognathism. He is alert and oriented to person, place, and time
. Chest auscultation is normal, as is heart examination. The rest of his physica
l exam is unremarkable. Monitoring with which of the following will help prevent
one of the major complications of this condition?

A.
Bone density
B.
Colonoscopy
C.
Prolactin levels
D.

Prostate-specific antigen
E.
Thyroid-stimulating hormone

The correct answer is B. Patients with acromegaly are more prone to developing c
olon cancer than the rest of the population, so monitoring with regular colonosc
opies is advised in patients diagnosed with acromegaly.
Bone density monitoring (choice A) is not necessary because these patients will
not have a higher incidence of osteoporosis than the population in general.
Prolactin levels (choice C) do not have to be monitored in these patients, even
though a patient with a macroadenoma that produces growth hormone and impinges o
n the pituitary stalk will have a higher level of prolactin than the general pop
ulation (and can manifest with amenorrhea and galactorrhea in females). Treating
acromegaly will treat this condition, however, so monitoring for hyperprolactin
emic states is not necessary.
Patients with acromegaly have no higher risk than the rest of the population of
developing prostate cancer, so PSA monitoring (choice D) is not recommended in t
hese patients.
Thyroid-stimulating hormone (choice E) does not have to be monitored in these pa
tients because their incidence of thyroid disorder is no higher than that of the
rest of the population.
A 39-year-old woman comes to the office because she has not had a menstrual pe
riod for the past 2 months. Before that time she was having regular monthly mens
trual cycles lasting 28 days with a 4-day menses. Lately she has been feeling un
usually cold and has been experiencing on and off constipation. She has a history
of migraines and had a laparoscopic left ovarian cystectomy 15 years ago. She is
a divorced mother of a 10-year-old daughter and 8-year-old son. She works as a
corporate lawyer and experiences a lot of stress. Physical examination shows puf
fy eyes, pale and dry skin, and diffuse alopecia. The remainder of the examinati
on, including breast and pelvic examinations, is normal. Which of the following
is the most appropriate next step in diagnosis?

A.
Endometrial biopsy
B.
Measurement of serum follicle-stimulating hormone level
C.
Measurement of serum thyroid-stimulating hormone level

D.
Measurement of urine human chorionic gonadotropin level
E.
Pelvic computed tomography scan

The correct answer is D. The most common cause of secondary amenorrhea in women
of childbearing age is pregnancy. It is absolutely essential to remember this. F
ailure to keep the possibility of pregnancy in mind when evaluating a patient ca
n lead to a failure to diagnose ectopic pregnancy or intrauterine pregnancy. Fai
lure to diagnose these conditions can lead to significant maternal morbidity and
mortality, fetal exposure to harmful medications or other exposures, and other
poor outcomes.
Endometrial biopsy (choice A) would absolutely not be the most appropriate next
step in diagnosis. Performing an endometrial biopsy without having a negative pr
egnancy test could lead to the inadvertent termination of a desired pregnancy. I
t is therefore essential to check a pregnancy test on this patient first.
Measurement of serum follicle-stimulating hormone level (choice B) is an appropr
iate step in the evaluation of amenorrhea. However, it is not the most appropria
te next step in this case because the first step in the evaluation of secondary
amenorrhea is to check a pregnancy test.
Measurement of serum thyroid-stimulating hormone level (choice C) is also an app
ropriate step in the evaluation of amenorrhea, inasmuch as hypothyroidism can le
ad to amenorrhea. However, although it is an appropriate step, it is not the mos
t appropriate next step in this case.
Obtaining a pelvic computed tomography (CT) scan (choice E) is not the most appr
opriate next step in diagnosis. First, the appropriate next step here is a pregn
ancy test. Second, CT is not a part of the routine diagnostic workup for seconda
ry amenorrhea.
A 51-year-old man comes to an urgent care clinic for evaluation of abdominal p
ain and swelling. He has noticed increasing abdominal girth over the last 3 to 4
months, as well as mild, diffuse abdominal pain. His past medical history is si
gnificant for alcohol abuse, complicated by an upper gastrointestinal bleed 2 ye
ars ago that is suspected to be from a tear of esophageal varices. Physical exam
ination reveals a cachectic, mildly jaundiced man in no apparent distress. Vital
signs are: temperature 37 C (98.6 F); blood pressure 110/80 mm Hg standing, 106
/78 mm Hg supine; pulse 88/min; respirations 18/min. He has a protuberant, diste
nded abdomen with shifting dullness. There is marked hepatosplenomegaly. Skin ex
amination reveals multiple telangiectasias on the patients chest wall and mild re
dness of the palms. Laboratory studies show:
Which of the following is the most appropriate initial treatment for this patien
ts hyponatremia?

A.
Fluid and sodium restriction
B.
Hyperosmolar (3%) saline
C.
Lactated Ringers
D.
Normal (0.9%) saline
E.
One-half normal (0.45%) saline

The correct answer is A. Hyponatremia is commonly seen in the edematous states o


f congestive heart failure, nephrotic syndrome, and, as in the case of this pati
ent, cirrhosis. The kidney, in a homeostatic response to reduced perfusion, reta
ins sodium (hence a concentrated urine with a low urine sodium) and water. Retai
ned sodium results in increased total body sodium, but relatively more intravasc
ular and interstitial water retention, resulting in hyponatremia. While volume r
estriction is not always the treatment for hyponatremia, it is a safe policy to
initially restrict fluid until the cause of the hyponatremia can be determined.
In this patient, the low serum potassium concentration indicates significant los
s of intracellular potassium content. Sodium will leave the extracellular fluid
to enter cells in order to maintain electrical neutrality, accentuating the hypo
natremia.
Hyperosmolar saline (choice B) is risky and appropriate only for symptomatic hyp
onatremia, which usually manifests as mental status caused by cerebral edema. Ra
pid development of hyponatremia does not allow cellular homeostatic mechanisms t
ime to adjust to the change in extracellular osmolality. The relatively hyperosm
olar intracellular environment is flooded with water, resulting in cellular swel
ling. Overcorrection with hyperosmolar saline can result in osmotic demyelinatio
n syndrome.
Lactated Ringers (choice C) and normal saline (choice D) would be excellent choic
es if this patient needed volume. However, there is no evidence of hypovolemia,
such as tachycardia or orthostatic hypotension. Volume given to this patient is
unlikely to stay in the intravascular space, and will likely just worsen this pa
tients ascites.
Half-normal saline (choice E) is a good choice for maintenance fluids to avoid h
ypernatremia. It will worsen existing hyponatremia, as it is of a lower osmolali
ty than blood.

A 26-year-old woman is being treated for type II diabetes. Treatment includes


monitoring of glucose levels, dietary change, an exercise regimen, and a new dia
betes drug. Over the past year, this treatment has successfully controlled her d
iabetes and the patient is happy with the outcome. One day you receive a phone c
all from another physician. The physician identifies herself as the physician of
the diabetic patients brother. The brother also is suffering from type II diabet
es. In contrast with his sister, his diabetes is uncontrolled. The brothers physi
cian says that he heard that the sister was on a treatment regimen that was work
ing and was calling to discuss the details of the treatment so that he could rec
ommend it to the brother. At this point, what action should the physician take?

A.
Offer to meet face-to-face with the brothers physician to discuss the sisters trea
tment regimen
B.
Offer to send a letter to the physician who called, detailing the treatment plan
for the sister
C.
Offer to send the physician literature provided by the pharmacology company abou
t the benefits and risks of this new drug
D.
Provide the details of the sisters treatment over the phone to the physician who
called
E.
Refuse to provide any information to the brothers physician without the sisters ex
plicit permission

The correct answer is E. Confidentiality is absolute. No information may be shar


ed unless the explicit approval of the patient is obtained first (choices A, B,
and C). Strictly speaking, the physician should not even confirm that the sister
mentioned here is his patient.
Even with the sisters permission, giving information about a patient to someone w
ho calls on the phone is a practice to avoid (choice D). The physician cannot be
sure of the identity of the caller and may end up, without knowing it, giving c
onfidential information to an inappropriate party.
A 71-year-old woman undergoes a total thyroidectomy and removal of a suspiciou

s jugular lymph node, which reveals lymphoma. The operation went well and there
was minimal blood loss. The patient was transferred to the recovery room and the
n to the surgical floor, where her recovery was originally unremarkable. However
, she is now quite agitated and anxious, and is complaining of tingling and numb
ness around her lips. A medical student who did a full history and physical thin
ks she might also have papilledema. Which of the following is an associated phys
ical finding likely to be found in this patient?

A.
An inspiratory pause when the right upper quadrant is palpated
B.
A large, nonreactive or only mildly reactive pupil
C.
Orbicularis oculi muscle spasm with gentle tapping of the facial nerve
D.
Pain induced by extension of the legs with hip flexed
E.
Flexion of the hips induced by passive flexion of the neck

The correct answer is C. This patient has acute hypocalcemia, likely from the su
rgeons not leaving one of the parathyroid glands during thyroidectomy. Acute hyp
ocalcemia can cause papilledema, neuropsychiatric changes including depression a
nd anxiety, and tetany. Tetany can be induced with gentle tapping of the facial
nerve. This will result in orbicularis oculi muscle spasm.
Murphy sign (choice A) is an inspiratory pause seen when the right upper quadran
t is palpated in patients with cholecystitis. As the diaphragm lowers, it displa
ces the inflamed gallbladder down toward the physicians palpating hand. The incre
ased pressure on the inflamed gallbladder will cause the patient discomfort, cau
sing the patient to stop inspiration.
Adie pupil (choice B) is the benign finding of a tonic pupil on physical exam, u
sually in younger women. The pupil will appear larger than its counterpart and w
ill be either nonreactive or only mildly reactive. It is of no clinical signific
ance and is not associated with hypocalcemia.
Brudzinski sign (choice E) and Kernig sign (choice D) are evidence of meningeal
irritation.
A 57-year-old woman comes to the clinic for a routine health check. She has no

current complaints but wants her blood pressure and her blood glucose level che
cked. Over the past 10 years, she has struggled with diabetes and hypertension.
She is assertive about preventive healthcare measures and has had recent normal
ophthalmologic examinations and podiatry visits. In addition to checking her dia
betes and hypertension and a urine microalbuminuria, you recommend a lipid panel
as part of her healthcare maintenance examination. Her lipid panel reveals a ch
olesterol level of 225 mg/dL, an HDL cholesterol level of 38 mg/dL, and an LDL c
holesterol level of 138 mg/dL. Which of the following is the goal LDL cholestero
l level for this patient?

A.
Depends on triglyceride levels
B.
Less than 100 mg/dL
C.
Less than 130 mg/dL
D.
Less than 160 mg/dL
E.
Less than 200 mg/dL

The correct answer is B. Diabetes is considered a coronary heart disease equival


ent. In large trials, it has been shown to be a significant risk factor for coro
nary events, and thus warrants aggressive LDL cholesterol control. The goal is l
ess than 100 mg/dL, and pharmacologic therapy is recommended if levels are great
er than 130 mg/dL. As diabetes is a major cardiac risk factor, lipid level check
s must be part of routine health care maintenance.
The evidence for serum triglycerides is not conflicting, as are recommendations
for treatment in patients with diabetes. LDL cholesterol treatment goals, howeve
r, do not depend on triglyceride levels (choice A). LDL cholesterol is an indepe
ndent risk factor for cardiac events.
Patients with two risk factors, which include hypertension, low HDL cholesterol,
tobacco use, family history, and age, have a lower LDL cholesterol goal. These
patients target LDL is less than 130 mg/dL (choice C).
For patients with no major risk factors for cardiac events, a goal of less than
160 mg/dL (choice D) is appropriate.
For any patient, a cholesterol level approaching 200 mg/dL (choice E) is too hig
h.

A 34-year-old woman is brought to the emergency department because of an episo


de of convulsions and coma that occurred about 45 minutes after she ate a very l
arge meal. On arrival, she is sweating profusely and barely waking up from her s
tuporous state. Blood is drawn for laboratory studies and she is taken to the CT
scanner for a CT of her head. The CT is completely normal. When the laboratory
studies come back they show her blood sugar to be 45 mg/dL. Insulin and C-peptid
e determinations are then requested, which eventually come back showing signific
ant elevations. After these finding, she is questioned further and gives a histo
ry that for many years she has felt faint after she eats. She describes episodes
where she has palpitations, sweating, weakness, and tremors, always half an hou
r to an hour and a half after she eats. If she skips meals for any reason, the s
ymptoms do not occur. Which of the following is the most appropriate next step i
n management?

A.
CT scan of the pancreas
B.
Psychiatric consultation
C.
Suitable dietary modifications
D.
24-hour urinary collection for catecholamines
E.
24-hour urinary collection for 5-hydroxy-indolacetic acid

The correct answer is C. The clinical picture here is classic for reactive hypog
lycemia, which follows food ingestion. Smaller and more frequent meals will amel
iorate the problem.
CT scan of the pancreas (choice A) would be appropriate if we suspected an insul
inoma, but such lesions invariably produce symptoms during fasting, and not afte
r meals.
Psychiatric consultation (choice B) would have been in order if we suspected sel
f-administration of insulin. But in that case, the C-peptide would have been low
at the same time that the insulin levels were high.
Catecholamines are indeed responsible for the sweating and palpitations that she
reports, but these are normal reactions to the hypoglycemia. Determining levels

in the urine (choice D) will not advance the diagnosis or therapy.


5-Hydroxy-indolacetic acid (choice E) is the marker for the carcinoid syndrome.
Hypoglycemia is not part of that disease. The attacks in the carcinoid syndrome
are due to circulating serotonin, and the classic findings are flushing, itching
, and diarrhea.
A 55-year-old woman with a long history of diabetes mellitus type 2 returns to
the clinic for a follow-up appointment. She was recently started on a new diabe
tes regimen after her last hemoglobin A1C was found to be over 10%. Today she is
complaining of shortness of breath, worse when she lies down, as well as swelli
ng in her legs. Her diabetes has been poorly controlled over the years, due part
ly to the severity of her disease and partly to her poor medical compliance. In
addition to her diabetes, she is followed in clinic for poorly compensated conge
stive heart failure, hypertension, and reflux disease. Examination reveals a thi
rd heart sound, wet-sounding crackles in both lung bases, and 3+ pitting edema b
ilaterally. Which of the following diabetes medications was most likely added to
her regimen that is responsible for these findings?

A.
Acarbose
B.
Glyburide
C.
Insulin glargine
D.
Metformin
E.
Rosiglitazone

The correct answer is E. The thiazolidinediones, such as rosiglitazone and piogl


itazone, are contraindicated in decompensated heart failure. These medications c
an cause fluid retention, a dilutional anemia, and precipitate frank anemia. Pat
ients need to be monitored closely for any evidence of heart failure. In additio
n to edema and heart failure, these medications can be hepatotoxic, requiring re
gular monitoring of liver function tests.
Acarbose (choice A) is an alpha-glucosidase inhibitor with minimal systemic effe
cts. Its main side effects are gastrointestinal discomfort, flatulence, and diar

rhea.
Glyburide (choice B) is a sulfonylurea, and is generally well tolerated. However
, because it works by increasing insulin secretion, it has a similar side-effect
profile. Weight gain and hypoglycemia are common adverse events.
Insulin (choice C) is unlikely to cause heart failure. The most common effects o
f insulin are weight gain and hypoglycemia.
Metformin (choice D) can cause a severe metabolic acidosis and should be avoided
in patients such as this one, who are predisposed to acidemia because of unstab
le heart failure.
A 65-year-old man comes to your clinic complaining of difficulty in maintainin
g an erection long enough for satisfying intercourse. He has no problems with li
bido or in attaining an erection and reports a happy marriage of 43 years. His d
ifficulties have been worsening steadily for the past 10 years. He has a history
of type 2 diabetes and hypertension that are well controlled on lisinopril and
glyburide. Examination reveals normally developed penis and testes. He has decre
ased pulses in his lower extremities bilaterally, and his neurologic examination
is unremarkable. Laboratory studies show:
Which of the following is the initial management most likely to be effective in
treating this patients erectile dysfunction?

A.
Change his oral hypoglycemic to metformin
B.
Prescribe intracavernous prostaglandin injections
C.
Prescribe oral sildenafil therapy
D.
Prescribe testosterone supplementation therapy
E.
Recommend intensive couples psychotherapy

The correct answer is C. This man has vasculogenic erectile dysfunction secondar
y to his hypertension and diabetes. Oral sildenafil now is considered first-line
therapy for men with organic erectile dysfunction (ED); doses of 100 mg restore

better quality erections in 81% of a general population of men with ED. The pil
l should be taken on an empty stomach 1 hour before desired intercourse. Sildena
fil is a selective inhibitor of phosphodiesterase type 5, which is primarily exp
ressed in the penis. By inhibiting phosphodiesterase, levels of cyclic GMP in th
e penis increase, causing relaxation of smooth muscle in penile arterioles. This
causes increased blood inflow, leading to firmer erections.
Neither of the medications he is taking is likely to cause erectile dysfunction,
so changing his hypoglycemic regimen (choice A) is unlikely to be of benefit. M
edications are an important consideration in evaluating ED, especially antihyper
tensives such as beta-blockers and diuretics. Antidepressants, paradoxically, al
so are strongly implicated in ED. Withdrawal of these agents as appropriate is u
sually the first step in therapy.
Intracavernous prostaglandin injections (choice B) are highly effective in helpi
ng to initiate erections, but these are also cumbersome to use and may cause pri
apism and cavernous fibrosis. For this reason, they are considered second- or th
ird-line agents.
Testosterone supplementation (choice D), although popular, has not been shown to
improve ED in men with normal testosterone levels. This patients normal levels o
f testosterone and LH (which stimulates testosterone production) rule out hypogo
nadism. Men with ED should be screened for low libido and have serum total testo
sterone and leuteinizing hormone levels checked. If the patient suffers from hyp
ogonadism, he should be started on a trial of transdermal testosterone or weekly
intramuscular testosterone depot injections before proceeding with further medi
cations.
For men with a significant component of psychogenic ED, couples therapy (choice
E) is an important adjunct to any therapy. Psychogenic factors likely do not pla
y a significant role in this case, although counseling may help him to overcome
any performance anxiety he may have developed in recent years. Oral sildenafil a
lso may be helpful to men with psychogenic ED, although at a lower rate.
A 68-year-old man comes to clinic demanding an erection pill. He states that ove
r the past few years he has had difficulty maintaining, though he is able to ini
tiate, an erection. He denies pain with ejaculation, premature ejaculation, or a
ny previous penile difficulties. He reports having a good relationship with his
spouse and denies any new stressors in his life, though his inability to maintai
n an erection is causing moderate distress. His past medical history is signific
ant for coronary artery disease, a myocardial infarction 2 years ago, stable ang
ina, diabetes, and benign prostatic hyperplasia. He currently takes a 5-alpha-re
ductase inhibitor, a biguanide, a beta-blocker, an ACE-inhibitor, a daily aspiri
n, and a long-acting nitroglycerin preparation. Physical examination reveals dim
inished pulses in the patients lower extremities and dystrophic toenails, but is
otherwise unremarkable. Which of the following is the most appropriate treatment
for this patients erectile dysfunction?

A.
Alprostadil
B.
Testosterone
C.
Sildenafil

D.
Vascular surgery
E.
Yohimbine

The correct answer is A. An effective intervention in patients with erectile dys


function who have a contraindication to sildenafil is alprostadil. Alprostadil i
s a prostaglandin that causes vasodilatation by direct vascular smooth-muscle re
laxation. Trabecular and cavernosal smooth muscle relaxes, allowing blood flow i
nto the lacunar spaces of the penis. It is given as a direct injection or transu
rethrally and has few systemic side effects, though a small number of patients e
xperience penile pain or priapism.
Testosterone (choice B) can be given to men with hypogonadism. However, a major
side effect of testosterone replacement is worsening benign prostatic hyperplasi
a, inasmuch as testosterone and its metabolites drive prostate growth. Patients
with severe BPH or with prostate cancer should not be given testosterone. Patien
ts with moderate BPH should be tried on other agents first. Further, although th
e etiology of this patients erectile dysfunction is not currently known, it is un
likely that hypogonadism is his primary problem. Up to 50% of diabetic men will
suffer erectile dysfunction, mainly from neuropathy but also from vascular disea
sewhich is quite likely the case with this patient.
Sildenafil (choice C), which increases nitric oxide levels, causes severe, lifethreatening hypotension when given with nitroglycerin derivatives. It is contrai
ndicated in this patient. If it is determined that he does not need nitroglyceri
n, then sildenafil is an excellent choice for this patient.
Vascular surgery (choice D) is a dramatic intervention in this patient and unlik
ely to be worth the risk. Vascular surgery for erectile dysfunction has poor res
ults in older men with generalized disease. It is generally reserved for young m
en with congenital or traumatic erectile dysfunction.
Yohimbine (choice E) is a common ingredient in over-the-counter enhancement produc
ts. It has not been proven effective in placebo-controlled trials, and is certai
nly not as efficacious as alprostadil.
A 29-year-old woman comes to your office because of fatigue and constipation.
She notes that these symptoms have been growing worse over the past few weeks. S
he also complains of cold intolerance, dry skin, and amenorrhea for the past 2 m
onths. She has no past medical history. Past surgical history is significant for
a cesarean delivery 2 years ago for a breech fetus at term. She takes no medica
tions and has no known drug allergies. Physical examination is unremarkable. Lab
oratory evaluation demonstrates a positive urine hCG and a thyroid-stimulating h
ormone (TSH) level of 10 mU/L. Free T4 is 0.2 ng/dL. Which of the following is t
he most appropriate next step in the management of this patient?

A.

Start levothyroxine, check TSH in 4 weeks


B.
Start levothyroxine, check TSH in 3 months
C.
Start propylthiouracil, check TSH in 4 weeks
D.
Start propylthiouracil, check TSH in 3 months
E.
No treatment is needed, recheck TSH in 3 months

The correct answer is A. This patient presents with the classic signs and sympto
ms of hypothyroidism: fatigue, constipation, cold intolerance, and dry skin. The
diagnosis of hypothyroidism can be missed easily during pregnancy, because the
signs and symptoms of hypothyroidism are ascribed instead to the pregnancy. It i
s important to identify and treat hypothyroidism during pregnancy as the disease
may progress to myxedema and myxedema coma if left untreated. Also, the materna
l symptoms often significantly worsen without treatment. Finally, and perhaps mo
st importantly, thyroid hormone is responsible for proper fetal development. Nor
mally there is no change in the TSH and free T4 levels during pregnancy. This pa
tient has symptoms together with an elevated TSH and low free T4. The diagnosis
of hypothyroidism therefore can be made and the patient started on levothyroxine
. It takes approximately 4 weeks for the levothyroxine to alter the TSH level. C
onsequently, TSH levels should be checked at 4-week intervals and treatment adju
sted accordingly until the TSH level is stable. Once the TSH level is stable it
may be checked each trimester. Levothyroxine requirements are expected to increa
se during pregnancy.
To start levothyroxine and check TSH in 3 months (choice B) would be incorrect b
ecause the TSH level should be checked approximately 4 weeks after a change in t
he dose of levothyroxine. To wait 3 months risks that the patient will be under
or over treated.
To start propylthiouracil and check TSH in 4 weeks (choice C) or to start propyl
thiouracil and check TSH in 3 months (choice D) would be incorrect because propy
lthiouracil should not be used to treat hypothyroidism. Propylthiouracil is used
to treat hyperthyroidism. This patient does not have hyperthyroidism. Her labor
atory findings of a high TSH and a low free T4 together with her symptoms point
to a diagnosis of hypothyroidism.
To state that no treatment is needed and recheck the TSH in 3 months (choice E)
is incorrect. Hypothyroidism should be treated during pregnancy because of the i
mpact that it can have on the mother (e.g., symptoms and myxedema) and the fetus
(i.e., fetal growth and development.)

A 34-year-old woman, gravida 2, para 2, comes to the office for a routine visi
t, 6 weeks after the delivery of a healthy baby. She states that she is feeling
well and has just gone back to work as a physicians assistant. Her pregnancy was
complicated by gestational diabetes controlled with insulin. She had a normal va
ginal delivery. She is currently taking no medications and has no drug allergies
. A 75-g, 2-hour oral glucose tolerance test is performed on the patient at this
6-week postpartum visit and it shows:
Which of the following is the most likely diagnosis?

A.
Diabetes mellitus
B.
Impaired fasting glucose
C.
Impaired glucose tolerance
D.
Persistent gestational diabetes
E.
Resolved carbohydrate intolerance

The correct answer is A. Diabetes mellitus has become an epidemic in the United
States. Increased rates of obesity and decreased rates of physical activity, alo
ng with the aging population, have combined to drastically increase the prevalen
ce of diabetes. Estimates are that women who have gestational diabetes mellitus
have a risk of 50% for eventually developing overt diabetes mellitus in their li
fetime. Therefore, the postpartum visit is a good opportunity to test for overt
diabetes those patients who had gestational diabetes during the pregnancy. This
can be done with a 2-hour plasma glucose evaluation. The American Diabetes Assoc
iation (ADA) in their Year 2000 guidelines uses the following criteria for the d
iagnosis of diabetes in nonpregnant persons: a plasma glucose level of more than
126 mg/dL after a fast of at least 8 hours; a plasma glucose level of more than
200 mg/dL 2 hours after an oral glucose tolerance test (75 g of glucose); or sy
mptoms consistent with the presence of diabetes, such as polyuria and polydipsia
, plus a plasma glucose level of more than 200 mg/dL, regardless of the time of

day at which the measurement was obtained. This patient has an elevated fasting
and 2-hour value and therefore, diabetes mellitus is the most likely diagnosis.
Note: The ADA guidelines do state that elevated fasting plasma glucose levels an
d results of the oral GTT should be confirmed by retesting on another day.
This patient has a degree of glycemia that goes beyond impaired fasting glucose
(choice B), which is defined as a fasting plasma glucose of 110 to 125 mg/dL.
This patients results from her 2-hour, oral glucose tolerance test also go beyond
impaired glucose tolerance (choice C), which is defined as a 2-hour plasma gluc
ose from 140 to 199 mg/dL.
The term persistent gestational diabetes (choice D) does not exist. Gestational di
abetes is defined as carbohydrate intolerance that begins or is first recognized
during pregnancy. A patient who continues to have glucose intolerance postpartu
m has diabetes mellitus.
This patient certainly does not have resolved carbohydrate intolerance (choice E
). On the basis of the results of her plasma glucose testing, she has diabetes m
ellitus.
A 66-year-old African American woman comes to your office after bone density t
esting reveals osteoporosis. She has hypothyroidism, for which she has taken thy
roxine for many years, but she has not kept regular appointments to follow up. S
he had a hysterectomy at the age of 43 and she has been on estrogen replacement
therapy since the age of 47. She takes a daily calcium supplement. She exercises
somewhat regularly. She does not smoke or use alcohol or drugs. Physical examin
ation, including breast and pelvic exams, is normal. Which of the following fact
ors is most likely adversely affecting bone mass in this patient?

A.
African American race
B.
Calcium supplementation
C.
Estrogen replacement therapy
D.
Exercise
E.
Thyroxine

The correct answer is E. Numerous factors affect a persons bone mass. It is impor
tant to recognize these factors and modify those that can be modified in order t
o prevent osteoporosis. Cigarette smoking, excessive alcohol use, and high caffe
ine intake appear to decrease bone mass and increase fracture risk. Dietary calc
ium intake is an important determinant of bone mass. It is essential to have ade
quate calcium intake from childhood onward. Vitamin D intake is also essential.
Family history is an important but unmodifiable risk factor. African American wo
men have a lower risk of osteoporosis than do white and Asian women. Genetic fac
tors also appear to play a significant role, with female children of women with
osteoporosis having lower bone mass compared with female children of women witho
ut osteoporosis. In terms of medications, estrogen, progesterone, calcium, and v
itamin D have all been shown to help increase bone mass. Thyroxine replacement t
hat suppresses thyroid-stimulating hormone (TSH) has been associated with decrea
sed bone density. This has been demonstrated at doses of 200 g or more daily. Sim
ilarly, hyperthyroidism itself is also associated with decreased bone mass.
African American women (choice A) are more likely to have increased bone mass an
d a lower risk of osteoporosis compared with women who are white or Asian.
Calcium supplementation (choice B) has been shown to be helpful in increasing bo
ne mass and decreasing bone loss. It does not adversely affect bone mass.
Estrogen replacement therapy (choice C) will help to prevent osteoporosis and bo
ne loss in women.
Exercise (choice D) has been shown to help improve bone density and to prevent o
steoporosis. The best exercise for bone strength is weight-bearing exercise such
as walking or jogging. Exercise is also beneficial because it helps people with
their balance and coordination. Improved balance and coordination are useful in
avoiding falls that can lead to hip and vertebral fractures.
A 35-year-old woman with a history of insulin dependent diabetes mellitus come
s to the clinic because she feels ill. She has no other medical issues and takes i
nsulin NPH 20 units twice daily. She has no allergies. She does not smoke and de
nies alcohol or drug use. Her blood pressure is 155/105 mm Hg and pulse is 80/mi
n. Lung, heart, and abdomen examinations are normal. Laboratory studies show:
A cortisol stimulation test is positive. After administration of 80 mg of furose
mide and 3 hours of upright posture, her plasma renin and aldosterone levels are
unchanged from baseline values. Which of the following is the most appropriate
treatment?

A.
Captopril
B.
Fludrocortisone
C.
Furosemide
D.
Hydrocortisone

E.
Potassium binders

The correct answer is C. This patient has hyporeninemic hypoaldosteronism that o


ccurs in adults with diabetes mellitus in the setting of mild renal failure, met
abolic acidosis and hyperkalemia. The defect is the result of hyporeninism. Usua
lly the aldosterone level rises after ACTH administration but not with postural
changes. In this patient, the furosemide will treat the hyperkalemia and the aci
dosis.
Administration of captopril (choice A) will worsen the hyperkalemia.
Administration of fludrocortisone (choice B) which is a potent mineralocorticoid
will help correct the electrolyte abnormality but should be avoided in patients
with hypertension.
Administration of hydrocortisone (choice D) is not indicated since the patient i
s not adrenally insufficient.
Administration of potassium binders (choice E) can enhance potassium excretion b
ut are likely to predispose the patient to total-body potassium deficits.
A 31-year-old pregnant woman at 28 weeks gestation comes to the office for foll
ow-up. On her previous visit, you had noted some weight loss despite a normal ap
petite. She also had a goiter and a new tremor. Laboratory studies show increase
d triiodothyronine and thyroxine levels, decreased thyroid-stimulating hormone l
evels, and the presence of thyroid-stimulating antibodies. Which of the followin
g is the most appropriate treatment at this time?

A.
Methimazole
B.
Propranolol
C.
Propylthiouracil
D.
Propylthiouracil and levothyroxine
E.

Radioactive iodine
F.
Subtotal thyroidectomy

The correct answer is C. The patient has Graves disease. Hyperthyroidism during
pregnancy is associated with Graves disease (most common cause), toxic multinodu
lar goiter, choriocarcinoma, hydatidiform mole, ovarian teratoma, and iatrogenic
thyrotoxicosis. Symptoms may include weight loss, tachycardia, exophthalmos, pr
etibial myxedema, generalized weakness, and tremor. Laboratory findings include
an elevated triiodothyronine, thyroxine, and low sensitive thyroid-stimulating h
ormone. Untreated hyperthyroidism in pregnant women is associated with an increa
sed incidence of neonatal thyrotoxicosis, spontaneous abortion, low birth weight
(premature delivery), preeclampsia, and congestive heart failure. Treatment dep
ends on the situation. Although both drugs cross the placenta, propylthiouracil
is preferred over methimazole because it does not cross the placenta as easily a
s methimazole does and is thus the drug of choice. Side effects of these medicat
ions include rash, urticaria, arthralgias, and agranulocytosis, which may predis
pose to maternal infection.
Close monitoring of thyroid hormones is required when administering antithyroid
medications during pregnancy. Levothyroxine (choice D) has no role.
Surgery (choice F) is reserved for severe refractive cases.
A 42-year-old man with a body mass index (BMI) of 28 and no significant past m
edical history is volunteering at a diabetes fundraiser where he is found to hav
e a fasting blood glucose level of 152 mg/dL. He had you repeat his fasting bloo
d glucose and it was 149 mg/dL. You have referred him to a dietitian for diabeti
c diet education, as well as advised him on an exercise program. The patient ret
urns to you 2 months later and states he has been compliant with the diet, is ex
ercising vigorously, and has lost about 5 pounds since his last visit. On physic
al examination, his vital signs are all within normal limits and his BMI is 26.
Complete physical examination is unremarkable. Laboratory studies performed 1 we
ek ago show:
Which of the following is the best next step in management?

A.
Continue diet and exercise alone
B.
Start insulin

C.
Start metformin
D.
Start a sulfonylurea
E.
Start troglitazone

The correct answer is D. It is important to understand the management of a newly


diagnosed noninsulin-dependent diabetic. In this case it is important to look at
body mass index (BMI) as a parameter to guide you in choosing the correct pharm
acologic agent for the initial treatment of diabetes. If the patients BMI is less
than 30, the initial drug of choice to start treating a diabetic patient who ha
s been recently diagnosed is going to be sulfonylureas.
This patient clearly needs a pharmacologic agent on top of diet and exercise bec
ause after 6 to 8 weeks his fasting blood glucose remains above 140 mg/dL. There
fore, continuing diet and exercise alone (choice A) would not be an appropriate
management plan.
This patient has only minimal elevation of his fasting blood glucose, which can
be controlled with oral hypoglycemic agents. Therefore, unless he is not respond
ing to oral hypoglycemic agents or his fasting blood glucose is markedly elevate
d, insulin (choice B) is not warranted.
Starting metformin (choice C) would be an appropriate choice in a patient whose
BMI is greater than 30; this patients BMI is below 30. The reason for using metfo
rmin and not sulfonylureas is that metformin produces gastrointestinal side effe
cts, including nausea, which cause the patient to lose weight (which can be bene
ficial in the treatment of a patient with diabetes who is also obese).
Troglitazone (choice E) was removed from the market in recent years due to its s
evere liver toxicity. There are still other drugs of the same family on the mark
et, but care should be taken to avoid severe liver damage from this type of drug
.
A 50-year-old man was recently diagnosed with diabetes mellitus type 2 after t
argeted screening revealed persistently elevated fasting blood glucose levels. S
ince being diagnosed, he has started metformin and an ACE inhibitor, and has his
statin therapy adjusted to reach a goal of an LDL-cholesterol of 100 mg/dL. Des
pite counseling, he continues to smoke, but is following a diabetic diet and exe
rcising regularly. He is currently healthy, and is up to date with his vaccinati
ons, having received a pneumonococcal vaccination five years prior, and an influ
enza vaccine earlier this year. Which of the following is an appropriate strateg
y for health care maintenance in this patient?

A.

Annual monitoring of hemoglobin A1C


B.
Dilated eye exams on a six-month basis
C.
Empiric use of daily aspirin therapy
D.
Nerve conduction screen for neuropathy
E.
Repeat pneumococcal vaccination yearly

The correct answer is C. Aspirin should be used for primary prevention in patien
ts with diabetes who do not have contraindications to the use of the medication.
A baby aspirin once daily is a simple, efficacious strategy to reduce the risk
of heart disease, and is recommended for diabetics, particularly if other cardia
c risk factors, such as smoking, are present.
Hemoglobin A1C (choice A) should be monitored every three to six months to ensur
e adequate glycemic control. As the natural progression of diabetes mellitus typ
e 2 is increasing medication requirements, it cannot be assumed that even compli
ant patients will have adequate glycemic control over a prolonged period of time
. Monitoring on a more frequent basis than annually is essential.
A dilated eye exam (choice B) by an ophthalmologist should be performed annually
. There is no need for more frequent eye examinations unless the patient has kno
wn disease or visual complaints.
Neuropathy (choice D) should be screened for at least annually, during the annua
l diabetic foot exam. Nerve conduction studies are not used to screen asymptomat
ic patients for neuropathy.
In general, the pneumococcal vaccination (choice E) does not need to be repeated
, though patients who receive the vaccine before age 65 may wish to have it repe
ated once they are over age 65.
A previously healthy 46-year-old woman comes to the office complaining of gene
ralized muscle weakness, increased urination and thirst, and headache. She denie
s fever, chills, weight loss, night sweats, flushing, dysuria, and hematuria. Pa
st medical history is significant only for normal, spontaneous, vaginal deliveri
es 17 and 20 years ago. She has no previous surgical history. She denies any abn
ormal menstrual cycles. She takes no medications and denies alcohol, tobacco, or
illicit drug use. Her temperature is 37.0 C (98.6 F), blood pressure is 154/102
mm Hg without orthostatic changes, pulse is 72/min, and respirations are 12/min
. She is in no distress. Her heart has a regular rate and rhythm without murmurs

. There are no palpable abdominal masses. Extremities have no ecchymoses, cyanos


is, clubbing, or edema. Complete blood count is normal. A CT scan of the adrenal
glands shows a 1-cm adrenocortical adenoma on the left side. Evaluation of her
serum electrolytes is most likely to show which of the following abnormalities?

A.
Decreased potassium, decreased sodium
B.
Decreased potassium, elevated sodium
C.
Decreased potassium, normal sodium
D.
Elevated potassium, normal sodium
E.
Normal potassium, elevated sodium

The correct answer is B. Conns syndrome is defined as adrenal hypersecretion of


aldosterone in the hypertensive, nonedematous patient. It accounts for approxim
ately 1% of the hypertensive patients in the United States. Women with this diag
nosis outnumber men by 2:1. The typical patient is 30-50 years old. Clinical sus
picion for primary hyperaldosteronism should be entertained in any hypertensive
patient with spontaneous hypokalemia (potassium <3.5 mEq/L). Patient complaints
include muscle weakness, polyuria, and headache. The headaches are secondary to
hypertension. Muscle weakness, polyuria, and paresthesias relate to the effect o
f hypokalemia on skeletal muscle, the renal concentrating mechanism, and periphe
ral nerves, respectively. Laboratory analysis shows hypokalemia, dilute urine wi
th pH >6.5, elevated serum bicarbonate, and mild metabolic alkalosis. Aldosteron
e is an important component of the rennin-angiotensin-aldosterone axis. Aldoster
one seeks to restore circulating blood volume. It does this by acting on the dis
tal tubule and collecting system of the kidney to cause secretion of K+ and, to
a lesser extent, H+ in exchange for sodium. The retained sodium osmotically caus
es water to be reabsorbed into the body. This results in hypokalemic, hypernatre
mic, metabolic alkalosis. Choices A, B, D, and E are therefore incorrect.
A 34-year-old woman is being evaluated for weight gain, depression and amenorr
hea over the past 7 months. She denies prior medical issues, takes no medication
s and has no allergies. Her blood pressure is 150/95 mm Hg. She is noted to be o

bese and she has dermal striae. As part of an evaluation she receives a midnight
dose of 1 mg of dexamethasone. Her 8 am cortisol level is low normal at 14 mcg/
dL. Which of the following is the most appropriate next diagnostic step?

A.
Abdominal computerized tomogram scan
B.
Pituitary gland computerized tomogram
C.
2-day high-dose dexamethasone suppression
D.
Urine 17-hydroxycorticosteroid level
E.
Urine 24-hour free cortisol level

The correct answer is E. This patient should be evaluated for Cushing syndrome.
The single dose overnight dexamethasone suppression test is a screening test in
the work up of cortisol excess. Suppression of plasma cortisol to less than 5 mc
g/dL implies normal hypothalamic-pituitary-adrenal feedback and excludes Cushing
syndrome as the diagnosis. Suppression failure is not diagnostic. A 24-h urine
free cortisol level is necessary to exclude or establish Cushing syndrome as the
diagnosis.
Imaging procedures should be performed only after a diagnosis of cortisol excess
has been established (choice A).
Pituitary scanning (choice B) should be deferred until chemical workup to evalua
te the hypothalamic-pituitary-adrenal axis has been completed.
Besides the 24-hour free urine cortisol level, a 2-day low dose dexamethasone su
ppression trial may be tried to evaluate for cortisol excess (choice C).
Late-onset congenital adrenal hyperplasia (choice D) is characterized by elevate
d urinary 17-ketosteroids and plasma DHEA sulfate.
A 28-year-old woman comes to clinic complaining of a new tremor. For the past
three weeks, she has suffered from a mild tremor, which seems to be fairly const
ant with activity or rest, and is somewhat worse at night. Additionally, she has
had an unexpected weight loss of 10 pounds during the last three weeks, a devel
opment she is not too displeased with. A complete review of systems reveals some
other expected findings; she reports diarrhea, insomnia, temperature intoleranc

e, and general irritability. She has a blood pressure of 142/90 mm Hg, pulse of
118/min, respirations of 20, and temperature of 37.8 C (100.0 F). Examination re
veals an exquisitely tender, diffusely enlarged thyroid. A mild tremor is presen
t when the patient extends her arms, but the rest of the exam is unremarkable. L
aboratory work-up is unremarkable, with the exception of thyroid function tests,
listed below:
The patient is started on a scheduled dose of NSAIDs every eight hours. Which of
the following is the most appropriate next test to order?

A.
Free triiodothyronine (T3) level
B.
No further tests; follow clinically
C.
Radionucleotide uptake scan
D.
Surgical consult for thyroidectomy treatment
E.
Ultrasound of thyroid gland

The correct answer is B. This patient likely has a transient thyroiditis, which
manifests as a tender thyroid with transient symptoms of hyperthyroidism. The di
agnosis is clinical. Aside from a baseline set of thyroid function tests, the pa
tient does not need an extensive workup. It the patient appears otherwise health
y, she can be followed clinically to ensure that her symptoms resolve. Therapy u
sually requires nothing more than NSAIDs.
A T3 level (choice A) is unlikely to be helpful. It will be mildly elevated, as
the inflammatory process associated with a tender thyroid results in the release
of colloid, which contains both T3 and T4 (though relatively more T4).
A radionucleotide uptake scan (choice C) can be useful in evaluating thyroid nod
ules. Hot nodules are more likely to be inflammatory, while cold nodules have a high
er likelihood of being malignant. However, this test is not necessary for a diff
usely enlarged thyroid that is tender.
Surgical resection (choice D) is not appropriate at this time. Tender glands are
unlikely to be malignant.
An ultrasound (choice E) can help define a suspected nodule or mass, and determi

ne if a lesion is cystic or not. It is less helpful if a thyroid is diffusely en


larged, particularly if the thyroid is tender, in which case the risk of a malig
nant lesion is low.
A 49-year-old woman comes to the clinic for a routine healthcare checkup. She
has a past medical history that includes diabetes and hypertension. Her diabetes
and hypertension have been well controlled, requiring metformin, lisinopril, an
d hydrochlorothiazide, in addition to the baby aspirin and potassium supplementa
tion she takes daily. She was discovered to have dyslipidemia 1 year earlier, ho
wever. Despite following a strict diabetic diet, exercising regularly, and takin
g a high-dose statin, her LDL-cholesterol remains at greater than goal. Addition
ally, her HDL-cholesterol is mildly less than normal. Which of the following is
an appropriate medication to start at this time?

A.
Additional statin
B.
Bile acid sequestrant
C.
Fish oil supplements
D.
Probucol
E.
Short-acting nicotinic acid

The correct answer is B. Bile acid sequestrants, such as cholestyramine, colesti


pol, or colesevelam, can be added to a statin when target goals have not been me
t. The addition of a sequestrant synergistically decreases LDL-cholesterol level
s (up to a 50% decline compared with no therapy), and modestly increases HDL-cho
lesterol levels. Choices other than a bile acid sequestrant include a fibrate or
a nicotinic acid derivative, though each is appropriate only in certain clinica
l situations.
An additional statin (choice A) is unlikely to provide increased benefit. The st
atin treatments differ in potency, and sometimes a patient responds to a change
in statin. Many patients with persistent dyslipidemia on treatment, however, req
uire an additional medication.
Fish oil supplements (choice C) help mainly with triglyceride levels and only mo

destly diminish LDL levels. Further, they can potentiate hyperglycemia and need
to be used cautiously (if at all) in patients with diabetes.
Probucol (choice D) is no longer available in the United States. It has been fou
nd to cause a decline in HDL-cholesterol levels believed to more than offset the
cardioprotective benefits of its moderate reduction in LDL-cholesterol.
Nicotinic acid (choice E) can be added to a statin to help reduce LDL-cholestero
l levels. In patients with diabetes, however, it can worsen glycemic control, as
the drug promotes insulin resistance. The short-acting versions are particularl
y difficult to use in patients with diabetes, whereas the long-acting crystallin
e versions are usually manageable.
A 38-year-old woman comes to the clinic for a routine checkup. Although she ha
s no specific complaints, review of systems reveals increasing weakness and fati
gue and a 30-lb weight gain since her last annual visit. Additionally, she repor
ts she notices that she bruises easily and suffers from menstrual irregularities
, problems that she never had before. Her past medical history has been unremark
able before this, with no major illnesses or chronic diseases. She has a tempera
ture of 37.0 C (98.6 F), blood pressure of 152/92 mm Hg, pulse of 78/min, and re
spirations of 20/min. Examination reveals a mildly obese woman in no apparent di
stress. Her pattern of fat distribution is odd, as there is accumulation in a ma
le truncal pattern, accumulation in her face, and on her upper back and neck. He
r extremities, however, appear thin and mildly atrophic. She has mild hirsutism,
acne, and thin, discolored skin on the dorsum of her hands. Large, violaceous s
triae are present on the abdomen. Laboratory studies show:
Which of the following is an appropriate test to confirm this womans diagnosis?

A.
Cosyntropin administration and serum cortisol monitoring
B.
Metyrapone stimulation test
C.
Overnight dexamethasone suppression test
D.
Secretin stimulation test
E.
Spot urinary and blood cortisol level

The correct answer is C. This patient has the classic signs, symptoms, and labor
atory abnormalities of Cushing syndrome, an overproduction of cortisol. Appropri
ate screening tests include an overnight dexamethasone suppression test and 24-h
our urinary cortisol level (as opposed to choice E, which looks at a spot blood
and urine level). One mg of dexamethasone is given in the evening and an 8 AM co
rtisol level is checked. If the dexamethasone suppresses cortisol production as
expected, Cushing syndrome is unlikely.
A cosyntropin stimulation test (choice A) is used in cases of suspected adrenal
insufficiency. Cosyntropin, a synthetic ACTH, is administered, and cortisol leve
ls are checked at 30 minutes and 1 hour after administration for an expected inc
rease in serum cortisol. If there is a significant response, adrenal insufficien
cy can be excluded.
The metyrapone stimulation test (choice B) is used to test for ACTH deficiency.
It works by inhibiting cortisol synthesis, which should result in an increase in
ACTH levels by reduced negative feedback. It is used to evaluate the hypothalam
ic-pituitary-adrenal axis and is not used for the diagnosis of Cushing syndrome.
A secretin stimulation test (choice D) is used to diagnose a gastrinoma. A gastr
inoma paradoxically increases secretion when secretin is infused, allowing the d
iagnosis of Zollinger-Ellison syndrome. This test is not used in the evaluation
of Cushing syndrome.
A 45-year-old man with a longstanding history of diabetes comes to the clinic
with increasing claudication. He also has known peripheral vascular disease. He
denies symptoms of chest pain or shortness of breath. He denies any other past m
edical history. He currently is receiving insulin. He has no known allergies. He
has documented vascular disease with a documented ankle the brachial index <0.90
. As part of his evaluation, his lipid profile is obtained. Which of the followi
ng information also should be obtained?

A.
C-reactive peptide
B.
Exercise stress tests
C.
Fasting glucose level
D.
Hemoglobin A1C
E.
Lipoprotein (a)

The correct answer is D. Given this patients known history of peripheral vascular
disease and diabetes, he needs intensive risk reduction. This involves optimizi
ng his lipid panel and hemoglobin A1C, which is an indication of his diabetic co
ntrol. Additionally, his blood pressure needs to be controlled.
C-reactive peptide (choice A) is gaining increasing recognition for its associat
ion with inflammation and cardiovascular disease. It is, however, a marker for d
isease. Elevated levels do not guide management.
Because this patient is not presenting with obvious signs of cardiac distress or
chest pain, an exercise stress test for risk stratification purposes is not req
uired (choice B).
This patient has known diabetes. Measurement of his fasting glucose level thus i
s not needed because the diagnosis is known already (choice C).
Lipoprotein (a) (choice E) also is gaining increasing recognition for its associ
ation with inflammation and cardiovascular disease. It is, however, a marker for
disease. Elevated levels do not guide management.
A 65-year-old woman complains of fatigue. She has no prior medical history and
is not taking any medications. She denies a recent weight change, heat or cold
intolerance, constipation, nausea, vomiting, dyspnea, chest pain, or muscle cram
ping. She has not noted blood in stool. She is able to perform activities of dai
ly living. Her blood pressure is 115/80 mm Hg and pulse is 65/min. Her neck is s
upple with no nodules; heart, lung and abdomen examinations are normal. Reflexes
are intact. Laboratory studies show a normal thyroid-stimulating hormone level,
a normal hematocrit, an elevated calcium level, a low phosphorus level and an u
ndetectable parathyroid hormone level. Which of the following is the most likely
diagnosis?

A.
Familial hypocalciuric hypercalcemia
B.
Hyperthyroidism
C.
Hypervitaminosis
D.
Malignancy
E.
Primary hyperparathyroidism

The correct answer is D. Patients presenting with hypercalcemia and hypophosphat


emia typically have a high parathyroid hormone level. Those with hypercalcemia a
nd hypophosphatemia without elevated levels of parathyroid hormone are likely to
have hypercalcemia of malignancy. This is associated with many solid tumors inc
luding lung and kidney cancer where a parathyroid-related protein is synthesized
by the tumors not identified by the currently available assays.
Familial hypocalciuric hypercalcemia (choice A) is an autosomal dominant trait i
n which patients often have normal or slightly low levels of parathyroid hormone
.
Hyperthyroidism (choice B) will cause hypercalcemia as a result of increased bon
e turnover. This is non-parathyroid hormone related and the phosphorus would not
be low. This patients thyroid-stimulating hormone is normal as well.
Hypervitaminosis (choice C) causes a non-parathyroid hormone mediated hypercalce
mia. The patients would not be expected to have a low serum phosphate.
Primary hyperparathyroidism (choice E) will cause hypercalcemia and hypophosphat
emia in the setting of an excess of parathyroid hormone activity.
A 69-year-old woman is brought by ambulance to the emergency room complaining
of severe muscle pains and cramps. She has a history of end-stage renal disease
due to long-standing hypertension and diabetes, and is well known to the emergen
cy room staff. Because she is often noncompliant with her medications and dialys
is treatment, she requires frequent hospitalizations. A quick review of her reco
rds indicates that she has missed her last two hemodialysis appointments, and ha
s a creatinine of 9.4 mg/dL. A stat electrocardiogram is ordered to gauge the se
verity of her electrolyte abnormality. Chemistries are pending. Which of the fol
lowing is the first change likely to be seen on the electrocardiogram in this pa
tients electrolyte abnormality?

A.
Diffuse P-wave flattening
B.
Prolonged PR interval
C.
Prominent U waves
D.
Sinusoidal-appearing QRS
E.
Tall, peaked T waves

The correct answer is E. Noncompliant dialysis patients with muscle cramps are o
ften suffering from hyperkalemia. The EKG findings of hyperkalemia are important
to recognize, and occur in stepwise fashion. The first finding is the presence
of tall, peaked T waves. As the hyperkalemia worsens, P-wave flattening (choice
A) becomes prominent. The PR interval then becomes prolonged (choice B) and the
QRS complex begins to widen. This finding presages life-threatening hyperkalemia
. In severe cases, the EKG will appear sinusoidal (choice D). Knowing the progre
ssion of EKG changes allows you to roughly gauge the urgency of intervention and
choose an appropriate treatment.
Prominent U waves (choice C) are present in hypokalemia. Patients with severe re
nal failure lose the ability to excrete potassium and are unlikely to suffer fro
m hypokalemia.
A 30-year-old man has chest pain and shortness of breath for the past 4 hours.
He denies prior episodes. He has no medical issues, takes no medications and ha
s no allergies. He reports a strong family history of coronary artery disease. E
valuation reveals positive cardiac enzymes, consistent with a non-Q wave myocard
ial infarction. He is admitted for management. Examination shows nodular swellin
gs on his Achilles tendon. His serum cholesterol level is 400 mg/dL. Which of th
e following protein defects is the most likely cause of this patients condition?

A.
Apoprotein CII
B.
Apoprotein E
C.
LDL receptor
D.
Lipoprotein (a)
E.
Lipoprotein lipase

The correct answer is C. Familial hypercholesterolemia is an autosomal dominant


defect in the LDL receptor. As a result, tissues are unable to take up cholester
ol rich LDL lipoprotein particles from the plasma. The high LDL levels lead to e
nhanced infiltration of cholesterol into the arterial wall causing enhanced athe
rosclerosis.
Apoprotein CII (choice A) deficiency leads to familial apoprotein CII deficiency
. This causes elevation in chylomicrons and VLDL.
Apoprotein E (choice B) defect causes familial type 3 hyperlipoproteinemia and a
n increase in chylomicron remnants. Treatment is with gemfibrozil.
Lipoprotein (a) is a particle that has been suggested to increase the risk of co
ronary artery disease when present in large amounts. It is nearly identical in s
tructure to LDL particles. The particle may slow the breakdown of blood clots (c
hoice D).
Familial lipoprotein lipase deficiency (type 1 hyperlipoproteinemia) is a rare a
utosomal recessive genetic disorder that results from a deficiency of lipoprotei
n lipase. The result is massive chylomicronemia (choice E).
A 46-year-old Hispanic woman with diabetes mellitus type 2 comes to the clinic
for a routine annual physical examination. She has no current complaints, has b
een compliant with her medications, and reports good glycemic control based on t
he results of her home glucose monitor. Her review of systems is unremarkable. S
ince her last visit, she has had no illnesses. She received an influenza vaccine
6 months prior and had a pneumonia vaccine 9 years ago. An ophthalmology appoin
tment is scheduled for next week, though she has not had any problems with her e
yes. She has never had a colonoscopy. Her past medical history, aside from diabe
tes, is significant for borderline hypertension and elevated cholesterol. Vital
signs today are: temperature 37.0 C (98.6 F), blood pressure 133/80 mm Hg, pulse
65/min, and respirations 20/min. Examination, including rectal and diabetic foo
t examination, is normal. A urinalysis dipstick is normal. Which of the followin
g is an appropriate health maintenance for this patient?

A.
Colonoscopy
B.
Pneumococcal vaccine
C.
Podiatry appointment
D.
Serum creatinine level
E.
Urine microalbumin level

The correct answer is E. Early diabetic nephropathy manifests with microalbuminu


ria (30 to 300 mg), which is below the level that can be detected with a urine d
ipstick. It is recommended that all patients with diabetes have an annual urine
microalbumin checked. If microalbuminuria is present, an ACE inhibitor should be
started. The progression of diabetic nephropathy can be slowed with aggressive
blood pressure control, an ACE inhibitor, and good glycemic control.
Colonoscopy (choice A) is not necessary until this patient is older than 50 year
s of age, after which time it should occur every 10 years, barring abnormal resu
lts.
A pneumococcal vaccine (choice B) should be given to patients with diabetes. It
does not need to be repeated until after age 65 years, at which time a booster s
hould be given.
Podiatry appointments (choice C) do not need to be scheduled annually if the pat
ient has no abnormalities on diabetic foot examination. If there are vascular co
mplications, however (e.g., dystrophic nails, foot infections), or neurologic pr
oblems (often manifesting as deformed joints), podiatry assistance can be invalu
able.
Serum creatinine (choice D) is an insensitive marker of early renal failure. It
does not need to be routinely followed, unless there is suspicion of renal failu
re or a change in the dose of an ACE inhibitor or diuretic.
A 65-year-old Hispanic woman has been treated successfully for diabetes over t
he past 5 years by a combination of dietary management and prescription medicati
on. Today, at a routine monitoring visit, the examination finds nothing out of t
he ordinary and you tell the patient everything looks good. I was wondering what yo
u would say, the patient replies with a smile, because I have not taken the medica
tion you prescribed for the past month, and I feel fine. She says that she is sti
ll watching her diet, but in place of the prescribed medication, she is taking a
n herbal remedy suggested by a local bruja. At this point, which of the followin
g is the best response?

A.
Tell me more about why you decided to make this change.
B.
Well, what youre doing seems to be working. Lets monitor you carefully and have you
back in here for a checkup in 2 weeks.
C.
What exactly is the herbal remedy you are taking?
D.

You are taking a big risk and I recommend that you go back to taking your medicat
ion at once.
E.
You have been lucky this month, but it is important for you to take your medicati
on as I prescribed it to maintain your health over the long term.

The correct answer is A. Before jumping to any conclusion, gather more informati
on. Find out why the patient made this decision and what prompted her action. Cl
early the patient did not feel comfortable discussing her decision to stop takin
g her medication with the physician before acting. This suggests some breakdown
in the trust of the physician-patient relationship that should be explored and a
ddressed. As a general rule, be accepting of folk medicine practices unless it i
s reasonable to suspect some probable harm or danger. A nonjudgmental tone is al
l-important. Notice with this reply the physician has not decided yet whether th
e patients action is harmful or actually may be acceptable. It is possible that d
ietary control is sufficient and she no longer needs to be on the prescribed med
ication.
Choices B and D are premature and close down a necessary discussion. The physici
an needs to find out more and explore why the patient did not say anything befor
e stopping her medication.
Asking detailed questions about the herbal remedy is inappropriate at this time
(choice C). Getting a sense of how the patient is thinking and why she took the
action she did is more important.
This implied threat may well express the physicians actual belief, but chides the
patient rather than exploring with her why she acted as she did (choice E). Get
more information before making pronouncements.
A 23-year-old woman was admitted 8 hours ago to the intensive care unit, where
you just started your shift, with a diagnosis of diabetic ketoacidosis. The pat
ient is alert and oriented to person, place, and time. She feels slightly weak a
nd complains of nausea and abdominal discomfort. She states she went on a weeken
d trip and forgot to take her insulin with her. Her blood pressure is 115/85 mm
Hg with no orthostatic changes. Her pulse is 85/min, respirations are 15/min, an
d temperature is 37.3 C (99.1 F). The rest of her physical examination is unrema
rkable. You have been ordering serial laboratory tests every hour, and the lates
t results obtained are:
The patient is currently on no food by mouth (NPO). She has good urine output an
d is on an insulin drip, receiving one-half normal saline solution as intravenou
s fluids with potassium supplementation. After you review the above lab findings
and the patients current treatment plan, which of the following is the best next
step in the management?

A.
Add 5% dextrose to the intravenous fluids

B.
Add phosphate to the intravenous fluids
C.
Give 20 units of insulin (NPH) subcutaneously and stop the insulin drip
D.
Hold potassium replacement in the intravenous fluids
E.
Start the patient on a clear liquid diet

The correct answer is D. In the management of DKA it is important to know the in


dication for potassium replacement. You should replace potassium in a patient wi
th DKA only if the potassium is less than 5 and the patient has good urine outpu
t. The patient in this case has good urine output but the potassium is 5.1. Ther
efore, potassium replacement should be stopped at this time.
This patients blood glucose is over 250; therefore, it would be inappropriate to
add 5% dextrose to intravenous fluids (choice A) at this time.
Adding phosphate to the IV fluids (choice B) is incorrect because phosphorus lev
els are normal so phosphate is not necessary at this time.
Stopping the insulin drip (choice C) is not appropriate in this patient at this
time because she still has positive acetone in serum.
Starting the patient on a clear liquid diet (choice E) is incorrect. This patien
t is still nauseated with abdominal discomfort so it would be inappropriate to s
tart feeding her at this time.
A 53-year-old woman comes to your office for advice regarding hormone replacem
ent therapy (HRT). She had her last menstrual period 1 year ago and since that t
ime has had intermittent hot flashes that disturb her sleep. She has occasional
migraine headaches and no other health problems. She had a left oophorectomy for
ovarian torsion at the age of 29 and has had no other surgeries. She takes no m
edications and has no known drug allergies. She states that she has heard so muc
h about HRT that she does not know what to think. She worries about breast cance
r, heart attack, and stroke, but she has heard conflicting reports about the eff
ect HRT can have on these diseases. She also worries about the hot flashes she i
s having. Which of the following is the proper counseling that should be given t
o this patient?

A.

HRT can be used to prevent breast cancer


B.
HRT can be used to prevent cardiac disease
C.
HRT can be used to prevent stroke
D.
HRT can be used to treat cardiac disease
E.
HRT can be used to treat hot flashes

The correct answer is E. Hormone replacement therapy (HRT) refers to the use of
estrogen and progestin in postmenopausal women. For many years it was thought th
at HRT had numerous health benefits for postmenopausal women, based on observati
onal studies. However, recently performed prospective, randomized-controlled tri
als have shown that HRT is associated with increased morbidity and mortality fro
m some conditions. Most notably, the Womens Health Initiative (WHI) Study demonst
rated that HRT was associated with increased rates of breast cancer, heart attac
k, stroke, and venous thrombosis. This finding led the United States Food and Dr
ug Administration (FDA) to require a black-box warning to be placed on all estroge
n-containing products for menopausal women. All estrogen-containing products for
postmenopausal women must now carry a black box warning, which is the strongest
type of warning on a drug label. The black box for estrogen-containing products
states that they should not be used for the prevention of heart disease. Estrog
en is indicated for the treatment of hot flashes, such as this patient has, but
the risks and benefits must be weighed by the physician and patient.
Counseling this patient that HRT can be used to prevent breast cancer (choice A)
would not be correct. HRT has now been shown to increase rates of breast cancer
. While there is still some controversy on this topic, it would not be advisable
to recommend that HRT be used to prevent breast cancer.
Advising this patient that HRT can be used to prevent cardiac disease (choice B)
is not correct. As stated earlier, on the basis of prior observational studies
it was previously thought that HRT would help to prevent cardiac disease. Given
the result of the randomized-controlled trials, however, this no longer appears
to be the case.
Telling this patient that HRT can be used to prevent stroke (choice C) would not
be correct. In randomized-controlled trials, women taking HRT were found to hav
e increased rates of stroke.
Stating that HRT can be used to treat cardiac disease (choice D) is not correct.
The evidence from available trials suggests that in women with preexisting card
iac disease, HRT causes increased morbidity and mortality from cardiac disease.

A 69-year-old man comes to clinic complaining of chronic pain in his joints. H


e is well known to the clinic, where he is followed for hypertension, diabetes,
and renal failure requiring hemodialysis three times per week. While he is compl
iant with dialysis, he often fails to take his medications, and tends to pick an
d choose which pills he takes. During the patients physical examination, multiple
small, firm subcutaneous nodules are noted on his forearms, chest, and back. Ad
ditionally, radiographs taken as part of the patients workup reveal diffuse calci
fications in the arteries, joints, and soft tissue. The pattern is described in
the radiologists report as a metastatic calcification pattern. Which of the followi
ng is the most likely underlying electrolyte abnormality?

A.
Hyperkalemia
B.
Hyperphosphatemia
C.
Hypocalcemia
D.
Hypokalemia
E.
Hypophosphatemia

The correct answer is B. Dialysis patients need to take phosphate binders. Phosp
hate is normally cleared by the kidney, but is poorly filtered by hemodialysis.
Long-standing hyperphosphatemia can result in calcium-phosphate aggregations, wh
ich diffusely precipitate, a process known as metastatic calcification. Further,
such patients are at risk for significant renal osteodystrophy. The homeostatic
response to the elevated PTH levels and calcium-phosphate aggregations is to re
lease more calcium from bone. As such, calcium levels are usually normal, not lo
w (choice C). As the kidney no longer clears phosphate, and dialysis does not re
move the phosphate, hypophosphatemia (choice E) is unlikely.
Patients who are compliant with dialysis usually do not suffer hyperkalemia (cho
ice A). Non-compliant dialysis patients with muscle cramps often suffer hyperkal
emia. As renal clearance of potassium is severely limited in renal failure, the
alternative situation, hypokalemia (choice D), is unlikely in renal failure pati
ents.

A 35-year-old woman has been complaining of severe headaches, with palpitation


s and sweating, which occur at random times during the day and last from 10 to 2
0 minutes. By coincidence, she has one of these episodes while waiting to be see
n by her physician, and the office nurse determines that her blood pressure at t
he time was 220/140 mm Hg. Subsequent testing shows elevated levels of vanillylm
andelic acid, norepinephrine and normetanephrine in a 24-hour urinary collection
. In that same test, levels of epinephrine and metanephrine are normal. CT scan
of her adrenal glands is likewise unremarkable. Which of the following is the mo
st appropriate next step in management?

A.
Alpha blockade and exploratory laparotomy
B.
Beta blockade and bilateral adrenal exploration
C.
Iodocholesterol scintigraphy (59 NP scan) of the chest and abdomen
D.
MRI of the adrenal glands
E.
Whole-body radioactively labeled I-metaiodobenzylguanidine (MIBG) scan

The correct answer is E. The clinical diagnosis is pheochromocytoma, but all ind
ications are that it is located at an extraadrenal site: the tumor makes only no
repinephrine, and it is not visible on CT of the adrenals. Pheochromocytomas are
typically quite large by the time they are symptomatic, and thus the negative C
T scan is significant. This tumor could be located anywhere from the neck to the
pelvis, and thus a whole-body scan is indicated with a substance (MIBG) known t
o be taken by chromaffin tissue in direct proportion to its biosynthesis of cate
cholamines. MRI can then be directed to the appropriate area for better visualiz
ation of the tumor.
Exploring the adrenal glands, either through the abdomen (choice A) or through t
he flank (choice B), would be inappropriate when all evidence points to an extra
adrenal tumor site. Additionally, choice B is flawed because it suggests operati
ng on a patient with a pheochromocytoma with only beta blockade can lead to seve
re, even lethal, hypertensive crisis as a result of unopposed alpha-adrenergic a
gonism.
The 59NP scan (choice C) is used to identify aldosterone-producing adenomas.

An MRI (choice D) is a better test than CT to distinguish pheochromocytoma from


benign adrenocortical adenomas, but MRI will not show a tumor that is not there
on the CT.
A 62-year-old man with coronary artery disease and atrial fibrillation is tran
sferred to the intensive care unit following a change in mental status. He was n
oted by his family to be progressively more lethargic over the last week. He has
been taking amiodarone for maintenance of sinus rhythm. He also has been treate
d with tamsulosin for benign prostatic hyperplasia. On examination, his rectal t
emperature is 30.6 C (87.0 F), blood pressure is 106/70 mm Hg, pulse is 37/min,
and respirations are 6/min. Heart and lung sounds are normal. His abdomen is sof
t. He is minimally responsive to painful stimuli. There is a delay in the relaxa
tion time of the deep tendon reflexes. Which of the following is the most approp
riate next step in management?

A.
Administer atropine, 1 mg intravenous bolus
B.
Administer triiodothyronine, intravenously
C.
Order STAT thyroid function tests
D.
Start transcutaneous pacing
E.
Warm the patient with a heating blanket

The correct answer is C. The most likely diagnosis is myxedema coma, which was c
onfirmed by the patients thyroid function tests. Further testing in this patient
revealed a urinary tract infection. Confirmation of the diagnosis is important,
as the therapy may have adverse effects such as myocardial ischemia, especially
triiodothyronine in patients with preexisting coronary disease.
As the patient is not hypotensive, there is no urgent need to reverse the bradyc
ardia. Giving atropine (choice A) and beginning transcutaneous pacing (choice D)
are therefore incorrect.
Administering triiodothyronine (choice B) is incorrect. Once the diagnosis has b
een established, administering levothyroxine or triiodothyronine is appropriate.
Saturation of thyroid hormone receptors should reverse the symptoms of myxedema

coma. Mortality is still high, however, even with therapy.


Vasodilatation by active reheating (choice E) may precipitate severe hypotension
in the hypothermia of myxedema coma. Such patients may be warmed passively.
A 50-year-old woman is brought to the emergency department by a friend in the
evening. Two days ago the woman began having symptoms of a urinary tract infecti
on, for which she saw a doctor and began taking antibiotics yesterday. However,
since then she has reported body aches, fatigue, and intolerance to cold; shed ha
d these for several months but they have recently worsened. The only medical his
tory known by the friend is an episode over a year ago when the patient had nipp
le discharge and subsequently had surgery for a tumor in her brain; she was on s
ome medications but stopped them when she ran out. The patient is able to corrob
orate some of this history but appears slightly confused and lethargic. She is t
hin and pale-appearing. She is afebrile, blood pressure is 105/65 mm Hg, pulse i
s 104/min, and respirations are 16/min. Physical examination shows dry mucous me
mbranes, nonpitting edema of the hands and lower extremities, and minimal body h
air. Reflexes are present but show slightly delayed relaxation. She has no evide
nce of hyperpigmentation. Laboratory studies show a sodium of 122 mEq/L, potassi
um of 3.6 mEq/L, chloride of 87 mEq/L, and bicarbonate of 27 mEq/L. BUN and crea
tinine are normal. Serum glucose is 80 mg/dL. Her leukocyte count is 14,000/mm3,
and her hematocrit and platelets are normal. Blood cultures are sent, and intra
venous fluids and antibiotics are administered. An ACTH stimulation test is plan
ned for the morning. In addition, which of the following should be the next adde
d therapy?

A.
Dexamethasone
B.
Fludrocortisone
C.
Glucose
D.
Hydrocortisone
E.
Levothyroxine

The correct answer is A. The most common causes of hypopituitarism usually relat

e to a neoplasm or a complication of a neoplasm, such as subsequent surgery or r


adiation. The manifestations depend on several factors, including the patients ag
e, the cell types affected, and the severity of each deficiency. Possible proble
ms include secondary adrenal insufficiency, central hypothyroidism, gonadatropin
deficiency, growth hormone deficiency, and prolactin deficiency. This patient i
s at risk for developing any of these complications after her surgery. The most
dangerous immediate problem is acute adrenal insufficiency, and thus glucocortic
oids are the treatment of choice. Because cortisol levels and response to ACTH s
timulation may need to be tested, dexamethasone is preferable; hydrocortisone
(choice D) is converted in the body to cortisol and would interfere with cortiso
l measurement. Hypothyroidism can be one of the manifestations of hypopituitaris
m; some of this patients manifestations are likely due to hypothyroidism. Even in
severe cases of hypothyroidism, however, if there is any suspicion for adrenal
insufficiency glucocorticoids should be administered first because thyroid repla
cement (choice E) can cause exacerbation of adrenal insufficiency.
Patients with primary adrenal insufficiency often manifest salt craving, hyperka
lemia, a metabolic acidosis, and, in severe cases, cardiovascular collapse, and
patients with secondary adrenal insufficiency have a deficiency of ACTH leading
to decreased cortisol secretion that does not usually affect aldosterone. Thus,
mineralocorticoid replacement, such as fludrocortisone (choice B), is usually no
t needed. Even in cases of primary adrenal insufficiency, the salt-retaining and
potassium-lowering effects often take days so mineralocorticoid replacement is
not the first drug given.
Glucose infusion (choice C) is incorrect because although this patients glucose i
s probably inappropriately low, given her current state and need to be watched c
losely, it is within the normal range and will improve with steroid administrati
on.
A 49-year-old diabetic man with end-stage renal disease comes to clinic compla
ining of bilateral leg and arm pain. He describes the pain as a diffuse, crampy
pain that is constant in nature. He has also been extremely weak, and states he
has trouble moving his lower extremities. The patient admits to having skipped h
is last two dialysis visits, as he has been depressed about needing to be hooked
up to a machine. Vital signs are: temperature 36.7 C (98 F), blood pressure 168/1
08 mm Hg, pulse 98/min, respirations 20/min. Physical examination shows periorbi
tal edema and 3+ pitting edema of the lower extremities. Neurologic examination
reveals diffuse muscle weakness with 4 out of 5 upper extremity strength and 3 o
ut of 5 lower extremity strength. Which of the following electrocardiogram findi
ngs is consistent with this patients most likely electrolyte disturbance?

A.
Diffuse ST elevation, diffuse PR segment depressions
B.
Flattened T waves, prominent U waves
C.
Large S wave in lead I, Q wave in lead III, and an inverted T wave in lead III
D.

Peaked T waves, wide QRS complexes, and loss of the P wave


E.
Short QT interval

The correct answer is D. Muscle cramps in a patient who misses dialysis is a cla
ssic presentation of hyperkalemia. Symptoms of hyperkalemia include muscle cramp
s, weakness, ascending paralysis, respiratory failure, and arrhythmias. EKG find
ings include progression from peaked T waves and a slightly widened QRS complex
to loss of P waves and a sinusoidal EKG tracing.
Diffuse ST elevation and diffuse PR segment depressions (choice A)are seen in pe
ricarditis. Pericarditis may be caused by uremia. However, this patient does not
have symptoms of pericarditis. Classic presentation would include a friction ru
b, positional chest pain, and a pleuritic component to the pain.
Flattened T waves and prominent U waves (choice B) are common findings of hypoka
lemia. Hypokalemia is commonly caused by excessive diuresis or dialysis.
The pattern of large S wave in lead I, Q wave in lead III, and an inverted T wav
e in lead III (choice C) is classic for pulmonary embolus. While it is highly sp
ecific, it is rarely seen. The most common EKG findings associated with pulmonar
y embolus are nonspecific ST segment and T-wave abnormalities, tachycardia, and
right axis deviation.
Short QT interval (choice E) as an isolated EKG finding is often associated with
hypercalcemia. There is no reason to suspect this patient has hypercalcemia.
A 38-year-old woman comes to the physician complaining of increasing weight lo
ss and anxiety. She had started a weight loss program approximately 6 months ago
and had been quite successful with the weight loss. She began to notice changes
in her body, however, and increasing weight loss after resumption of a regular
diet. In particular, she noticed that her hair has become thinner over the last
6 months and has started to gray quite rapidly. She notes that she has become qu
ite sensitive to heat and prefers cool environments; she also finds herself swea
ting much more often that she has in the past. She also has noted that she has b
een unable to sleep regularly, finding herself sleeping 2-4 hours a night. On re
view of systems, she notes that her heart occasionally races. She also has noted
that her menstrual flow has lightened. She denies any medication or substance u
se and denies any additional or increased stress in her personal or professional
life. Her vital signs are: temperature 37.2 C (99.0 F), blood pressure 128/56 m
m Hg, pulse 110/min, and respirations 16/min. Physical examination shows a promi
nent stare with a lid lag and noted conjunctival injection. She has a diffusely
enlarged anterior neck mass with a bruit heard over the area when auscultated. T
here is a 2/6 systolic flow murmur, fine, graying hair with small patches of alo
pecia on the scalp, thickened, reddish skin on the patients anterior legs, and se
paration of her nails from her nail bed. She has a mild tremor and weakness in h
er proximal arms and thighs. Which of the following findings is most likely to b
e decreased?

A.

T3 resin uptake test


B.
Thyroid radioactive iodine uptake
C.
Thyroid stimulating hormone
D.
Thyroid stimulating immunoglobulins
E.
Triiodothyronine (T3) and thyroxine (T4) levels

The correct answer is C. This woman has the classic history, signs, and symptoms
of Graves disease (diffuse toxic goiter). A triad of hyperthyroidism with a go
iter, ocular pathology (usually exophthalmos), and dermatologic findings (nail f
indings, pretibial myxedema), characterizes Graves disease. Patients often have
symptoms of hyperthyroidism that classically include weight loss, heat intolera
nce, skin changes, and resting tremor. In addition, the presence of goiter with
bruit on physical examination and proximal muscle weakness also lends credence t
o a hyperfunctioning thyroid gland. Usually the diagnosis can be made by using t
he triad criteria, but most often, diagnosis is confirmed with thyroid functioni
ng tests, showing very low levels of thyroid stimulating hormone and high levels
of T3 and T4 and resin T3 uptake.
T3 resin uptake test(choice A) determines the distribution of radiolabeled T3 be
tween an absorbent resin and the unoccupied thyroid hormone binding proteins in
the patients plasma. the binding of the labeled T3 to the resin is increased wh
en there is reduced unoccupied protein binding sites (e.g., TBG deficiency) or i
ncreased total thyroid hormone in the patients plasma; it is decreased under th
e opposite circumstances.
Graves disease is characterized by an enlarged gland and increased tracer uptak
e that is distributed homogeneously. Toxic adenomas appear as focal areas of inc
reased uptake, with suppressed tracer uptake in the remainder of the gland. In t
oxic multinodular goiter, the gland is enlarged-often with distorted architectur
e- and there are multiple areas of relatively increased or decreased uptake (cho
ice B).
If there is uncertainty about the nature of precipitating factors, the pathogeni
c process leading to hyperthyroidism is no longer in doubt. This depends on the
presence of antibodies directed against the thyroid follicular cell surface memb
rane TSH receptor; they are usually of the IgG1 subclass and in binding to the T
SH receptor activate the adenylate cyclase system, and thus stimulate thyroid fu
nction in vitro and in vivo. The levels of antibody correlate with thyroid funct
ion, and in humans the innoculation in vivo with purified antibody preparations
leads to increased thyroid activation; and in the offspring of mothers with high

levels of antibodies, passage of these antibodies across the placenta leads to


transient neonatal hyperthyroidism (choice D).
Triiodothyronine (T3) and thyroxine (T4) levels (choice E) are elevated in Grave
s disease and in most hyperthyroid, hyperfunctioning states, causing the symptom
s seen in this patient.
A 29-year-old woman comes to your office for follow-up of hypothyroidism and g
estational diabetes. She had a vaginal delivery 10 weeks ago, and during pregnan
cy her thyroxine was increased. She also had gestational diabetes during pregnan
cy, which was diagnosed with a 28-week glucose tolerance test. She was managed w
ith insulin during the pregnancy. She failed to keep her postpartum appointment
with her obstetrician. Which of the following is the most appropriate management
, given this patients history of gestational diabetes?

A.
75-g, 2-hour oral glucose tolerance test
B.
No medication or testing is needed
C.
Start glyburide treatment
D.
Start insulin treatment
E.
Start metformin treatment

The correct answer is A. Gestational diabetes is defined as carbohydrate intoler


ance that begins or is first recognized during pregnancy. Reported prevalence in
the United States ranges from 2 to 5%. However, the prevalence varies tremendou
sly on the basis of specific population or ethnic group studied. Women with gest
ational diabetes have an increased risk of developing the hypertensive disorders
of pregnancy. Offspring of women with gestational diabetes have increased rates
of macrosomia and hyperbilirubinemia. Most relevant to this patient, women with
gestational diabetes are at high risk of developing overt diabetes later in lif
e. Estimates are that as many as 50% of women with gestational diabetes will go
on to eventually develop overt diabetes mellitus, compared with only 7% of women
who do not have gestational diabetes. To detect those women who may have overt
diabetes, the American College of Obstetricians and Gynecologists recommends tha

t women with gestational diabetes undergo a 75-g, 2-hour oral glucose tolerance
test (OGTT) at the 6-week postpartum visit. This patient missed her postpartum v
isit; therefore, it would be appropriate to perform the OGTT at this time to det
ect whether or not this patient has diabetes mellitus, impaired fasting glucose,
or impaired glucose tolerance.
Stating that no medication or testing is needed (choice B) is incorrect. This pa
tient had gestational diabetes during her most recent pregnancy. She missed her
postpartum visit. She should, therefore, have an OGTT to determine whether or no
t she has diabetes.
Starting glyburide treatment (choice C), insulin treatment (choice D), or metfor
min treatment (choice E) would not be correct at this time. All of these medicat
ions can be used to treat patients with diabetes. Patients with gestational diab
etes are most often treated with insulin during the pregnancy because oral antid
iabetic agents are still considered to be experimental. This patient is no longe
r pregnant, so these drugs could be prescribed. However, it is important to note
that she has not been diagnosed with diabetes. She did have gestational diabete
s, but it is not clear at this time whether she has diabetes mellitus. Therefore
, the 75-g, 2-hour glucose tolerance test should be given to this patient first.
If she is found to have diabetes mellitus, then a treatment regimen can be sele
cted.
A 24-year-old woman comes to clinic complaining of a painful lump in her neck
and a new tremor. For the past three weeks, she has suffered from tremulous hand
s, which seems to be fairly constant with activity or rest, and is somewhat wors
e at night. About the same time as the tremor started, she noticed a small, pain
ful lump in the front of her neck. While she denies any dysphagia or odynophagia,
the mass hurts more when she yawns or extends her neck. Additionally, she has ha
d an unexpected weight loss of 10 pounds during the last three weeks, a developm
ent she admits she is not too displeased with. A complete review of systems reve
als some other interesting findings; she reports diarrhea, insomnia, and general
irritability. Vital signs are: blood pressure 145/85 mm Hg, pulse 114/min, and
respirations 22/min. Examination reveals an exquisitely tender, diffusely enlarg
ed thyroid, which is slightly asymmetric. A mild tremor is present when the pati
ent extends her arms, but the rest of the exam is unremarkable. Thyroid function
tests confirm a hyperthyroid state, with an elevated free thyroxine level and a
low thyroid-stimulating hormone level. The patient is started on NSAIDs, and, o
ver the next three weeks, her symptoms improve. Which of the following findings
is most likely associated with her condition?

A.
Antecedent acute or subacute bacterial infection
B.
Erythrocyte sedimentation rate within normal rangecream
C.
Infiltrative dermopathy in the skin overlying the shins
D.
Prodrome of fever, fatigue, malaise, and myalgias

E.
Strong family history of thyroid disease

The correct answer is D. This patient has suffered from a subacute thyroiditis,
also known as de Quervains thyroiditis. The typical triad includes neck pain, dif
fuse goiter (though nodules and asymmetry may be present), and a hyperthyroid st
ate that commonly lasts for 2 to 6 weeks. The condition is often post-viral, as
is commonly proceeded by or associated with fever, fatigue, malaise, and myalgia
s.
An antecedent bacterial infection (choice A) is unlikely; most cases of subacute
thyroiditis are felt to be either idiopathic or viral related. In severely immu
nocompromised hosts, bacterial infections should, however, be included in the di
fferential diagnosis.
The ESR is usually elevated, not normal (choice B). The pathophysiology of this
condition involves inflammatory damage to the thyroid, which induces colloid lea
k. The inflammatory response results in an elevated ESR.
Pretibial myxedema is an infiltrative dermopathy most commonly seen in the skin
overlying the shins (choice C). It is associated with Graves disease, and is an e
xtremely unlikely finding in subacute thyroiditis.
A family history of thyroid disease (choice E) is unlikely. While there are HLA
associations with de Quervains thyroiditis, it is not a familial disease.
A 40-year-old African American patient of yours has been extremely worried abo
ut diabetes, as both his mother and father passed away from complication of diab
etes. An overnight fasting blood glucose is 139 mg/dL. A repeat level 1 week lat
er, also fasting, is 129 mg/dL. The patient initiates a diet and exercise routin
e that results in the loss of 9 pounds over 3 months. On repeat testing, however
, his fasting blood glucose is 141 mg/dL. At this time, he denies any polyphagia
, polydipsia, polyuria, or visual changes. The rest of his review of systems is
likewise unremarkable. Past medical history is significant only for mild obesity
and borderline hypertension. Vital signs and physical examination are normal. A
t this time, which of the following is an appropriate course of action?

A.
Annual blood glucose monitoring
B.
Begin NPH insulin twice a day
C.
Increase diet and exercise goals

D.
Oral glucose tolerance test
E.
Start metformin

The correct answer is E. This patient meets diagnostic criteria for diabetes, as
he has persistently elevated blood glucose levels. There are three agreed upon
criteria for diagnosing diabetes mellitus type two. Meeting any one of the three
constitutes a diagnosis of diabetes. They are: (1) fasting blood glucose of gre
ater than 126 mg/dL on repeat tests, (2) abnormal oral glucose tolerance test, a
nd (3) signs and symptoms of hyperglycemia with a blood glucose level greater th
an 200 mg/dL. An appropriate initial therapy, if diet and exercise fail to contr
ol blood glucose, is to start an oral medication. Metformin often is chosen as a
first-line medication as it has the lowest risk for hypoglycemia, and, unlike m
any of the other agents, does not increase a patients weight.
Annual blood glucose monitoring (choice A) is inappropriate. This patient has di
abetes and needs frequent checks until glycemic control is achieved. Once achiev
ed, hemoglobin A1C levels, which give an idea of the average glucose levels over
the past 3 months, can be used to monitor glycemic control.
NPH insulin (choice B) is rarely used as a first-line agent. Insulin has the hig
hest risk for hypoglycemia and often results in weight gain that reduces periphe
ral muscle tissue glucose sensitivity. Furthermore, patients often are not incli
ned to have daily injections if they can avoid them.
It is not reasonable to expect more results from diet and exercise (choice C) in
this patient. Most results are seen in the first 5 to 10 pounds of weight loss,
which has not helped this patient. Furthermore, the loss of 9 pounds is a signi
ficant achievement, and more than most people can reasonably expect to lose and
keep off. After a 3-6-month period of diet and exercise, it is recommended that
medication be started in addition to lifestyle modifications.
An oral glucose tolerance test (choice D) is not necessary, as this patient alre
ady meets diagnostic criteria for diabetes. The results of such a test will cert
ainly be abnormal.
A 38-year-old Caucasian nurse comes to your office for her annual physical. Si
nce you saw her last, 1 year ago, she appears thinner, and the chart confirms a
weight loss of about 8 kg (18 lb). On questioning she admits to feeling anxious
on a regular basis and says the quality of her sleep has diminished. She also re
ports that she has had irregular and widely spaced menses in nearly 9 months. Ph
ysical examination reveals a thin woman who appears mildly nervous. She has a wi
de-eyed stare and moist skin. Her thyroid is nonpalpable, and cardiovascular exa
mination reveals tachycardia and a 2/6 early systolic murmur. Her deep tendon re
flexes are 3+. Serum levels of thyroid-stimulating hormone are undetectable and
serum thyroglobulin is low-normal. There is decreased radioiodine uptake. If she
continues in her current substance abuse, which of the following complications
could be expected?

A.
Exophthalmos
B.
Increased rate of bone loss
C.
Peptic ulcer disease
D.
Permanent goiter
E.
Premature ovarian failure

The correct answer is B. This woman has thyrotoxicosis factitia, or self-adminis


tration of thyroid hormone for purposes of weight loss. Such abuse of the levoth
yroxine is common among female health professionals. Aside from the cardiovascul
ar risks of prolonged hyperthyroidism (increased risk of sudden cardiac death, a
trial fibrillation, and exacerbation of mitral valve prolapse), one would expect
numerous other complications if the patient continues abusing the medication. T
he most problematic of these is an increased rate of bone loss in premenopausal
women, predisposing to spine, wrist, and hip fractures later in life. Bone loss
is caused by an increase in bone resorption, a decrease in the conversion of cal
cidiol (25-hydroxyvitamin D) to calcitriol (1,25-dihydroxyvitamin D), and an inc
rease in the clearance of calcitriol from the blood. This decrease in the level
of active calcitriol leads to decreased calcium absorption from the gut and an i
ncrease in calcium excretion in the urine. The overall effect is an accelerated
loss of both cortical and trabecular bone, which is already likely to be a probl
em in this thin Caucasian woman. Unfortunately, even if she stops taking levothy
roxine she is unlikely to be able to regain the bone mass lost during this episo
de.
Exophthalmos (choice A) is a complication of Graves disease, the most common cau
se of endogenous hyperthyroidism. Although a wide-eyed stare and lid lag are see
n in all forms of hyperthyroidism, exophthalmos (true protrusion of the globe of
the eye) occurs only in Graves disease and would therefore not be expected in t
his patient. Exophthalmos is independent of the autoimmune disease of the thyroi
d and is caused by a lymphocytic infiltration of the extraocular muscles. This p
rocess continues even after patients with Graves disease have had their thyroids
ablated.
Peptic ulcer disease (choice C) might be expected because this patient appears c
hronically stressed from her hormone overdose. Although peptic ulcer disease is
common and might occur independently, it has not been noted as a prominent compl

ication of chronic thyrotoxicosis.


Permanent goiter (choice D) would not be expected in a patient who chronically i
ngests excess thyroid hormone. In fact, levothyroxine will suppress TSH secretio
n from the pituitary, resulting in lack of stimulation of the thyroid and thyroi
d atrophy.
Premature ovarian failure (choice E) might appear to be occurring in this oligom
enorrheic patient. Oligomenorrhea is a common manifestation of thyrotoxicosis an
d is caused by a decrease in levels of free estradiol as well as a blunting of t
he midcycle LH surge. This leads to anovulatory cycles and exacerbates the incre
ased rate of bone loss noted earlier. However, the changes are reversible upon w
ithdrawal of the exogenous thyroid hormone, without permanent damage to the ovar
y.
A 26-year-old man is brought to the emergency department after being found by
his family on his bathroom floor. He had apparently collapsed after being increa
singly ill. His mother reports that over the last several weeks he has had sever
e insomnia, tremors, diarrhea, and fatigue. On arrival is only vaguely interacti
ve and he does not follow commands appropriately. His blood pressure is 143/89,
pulse is 125/min and his oxygen saturation is 96% on room air. On examination, h
e is diaphoretic and has a wide-eyed stare. He has a 2/6-midsystolic murmur at t
he second left intercostal space. His deep tendon reflexes are brisk throughout.
Laboratory studies show:
Which of the following is the most appropriate next test to establish the etiolo
gy of this patients collapse?

A.
24-hour urine for total catecholamines
B.
CT scan of the head
C.
Radioactive iodine uptake scan of the thyroid
D.
Serum thyrotropin-releasing hormone (TRH)
E.
Serum free triiodothyronine (T3) level

The correct answer is E. This patient is suffering from a severe bout of thyroto
xicosis, which is confirmed by his undetectable level of TSH. The surprising asp
ect of his presentation is the normal serum free T4. The most common causes of h
yperthyroidism, Graves disease and toxic multinodular goiter, both result in gr
eatly increased production of T4. However, they can often have a proportionately
greater increase in T3. Increased peripheral conversion of T4 to T3 may also pl
ay a role. In this patients case, a solitary hyperfunctioning thyroid adenoma i
s secreting mostly T3, which is much more potent on target tissues than is T4. T
his condition, called T3 thyrotoxicosis, often occurs in the initial stages of h
yperthyroidism, when symptoms are mild. However, it can also produce florid symp
toms such as those seen above. The most appropriate next test to establish the e
tiology of this patients collapse is the serum free T3 level.
A 24-hour urine for total catecholamines (choice A) is an appropriate initial te
st to screen for pheochromocytoma. Patients with these small, active tumors can
present with tachycardia, mental status changes, and autonomic hyperactivity. Ho
wever, their serum TSH levels will be normal, unlike this patient. The presence
of a depressed TSH in the presence of hyperthyroid-like symptoms should always m
ake the clinician suspect thyroid disease first.
A CT scan of the head (choice B), might be an appropriate study given his mental
status changes. However, given what we know about the course of his illness and
his laboratory results, imaging is unlikely to provide additional diagnostic in
formation.
Radioactive iodine uptake scan of the thyroid (choice C), is a common test to ev
aluate the cause of hyperthyroidism. However, it often adds little to the histor
y and physical examination and only serves to determine the appropriate dose for
radioiodine ablation of the thyroid. For example, patients with Graves disease
will often have a family history of thyroid disease and will have a diffusely e
nlarged goiter. Patients with a solitary hyperfunctioning adenoma and such flori
d symptoms will invariably have a palpable nodule on physical examination. In th
is patients case, the blood test is required first to confirm the diagnosis, an
d time cannot be spared to wait for a test as cumbersome as a thyroid uptake sca
n.
Serum TRH level (choice D), is rarely, if ever, useful in the evaluation of thyr
oid disease. The combined use of TSH, T4, and T3 studies makes this assay unnece
ssary, even when evaluating for "tertiary" or hypothalamic hyperthyroidism.
A 20-year-old woman with insulin-dependent diabetes comes to the office for a
scheduled followup appointment. She was diagnosed with diabetes at age 5 years a
nd has managed her illness very effectively. She has never had diabetic ketoacid
osis or hypoglycemic episodes. She denies fever, chills, weight loss, polyuria,
or excessive thirst. She has no other medical issues. Her medications include NP
H, 20 units twice daily and regular insulin, 5 units before each meal. Review of
her blood glucose record indicates that blood glucose levels are routinely 120150 mg/dL before breakfast, dinner, and bedtime, with the normal being 116 mg/dL
. Which of the following is the next most appropriate step in management?

A.
Decrease the dosage of NPH
B.
Decrease the dosage of regular insulin

C.
Increase the dosage of NPH
D.
Increase the dosage of regular insulin
E.
Make no changes and obtain a hemoglobin A1C

The correct answer is E. The patient has very good glycemic control. She may be
experiencing higher levels with snacks, however. Her near perfect sugar levels s
hould be confirmed objectively by checking a hemoglobin A1C level, which is an a
verage glucose level over the past 3 months.
Decreasing the NPH dosage (choice A) might increase sugar levels 4-6 hours later
.
Decreasing the dosage of regular insulin (choice B) would lead to an increase in
sugar levels 30 minutes later.
Increasing the NPH dosage (choice C) would decrease sugar levels 4-6 hours later
.
Increasing the regular dosage would decrease sugar levels 30 minutes later (choi
ce D).
A 46-year-old woman comes in for a followup visit. She has been checked every
year since she had a pheochromocytoma of her left adrenal gland successfully rem
oved at the age of 36 years. Her blood pressure is normal, she gives no recent h
istory of palpitations, headache, or perspiration, and she has no complaints. On
physical examination, a 2-cm hard nodule is felt in the right lobe of her thyro
id gland that was not present a year ago. There is no cervical adenopathy. Which
of the following laboratory studies would be most helpful at this time?

A.
Serum calcitonin
B.
Serum renin and angiotensin
C.
Thyroid stimulating hormone (TSH)

D.
Thyroxin (T4)
E.
Urinary catecholamines

The correct answer is A. Given the past history, the obvious concern is medullar
y carcinoma. Calcitonin is likely to be elevated, confirming the diagnosis. If t
he patient has the Sipple syndrome, there also may be elevations of parathyroid
hormone, ACTH, vasoactive intestinal polypeptide, prostaglandins, kallikreins, a
nd serotonin, none of which were offered as options, leaving calcitonin as the o
nly correct answer.
Renin and angiotensin (choice B) are not abnormal in medullary cancer, and neith
er is TSH (choice C) or T4 (choice D). Although presenting clinically as a thyro
id nodule, medullary cancer is not a tumor of thyroid tissue and it does not aff
ect thyroid function.
A thyroid mass in someone who had a pheochromocytoma is not suggestive of either
metastasis or recurrence of the original tumor, and thus there is no need to pu
rsue catecholamines (choice E) to ascertain the nature of the thyroid mass.
A 67-year-old man comes to your office complaining of weight gain, lethargy, a
nd cold intolerance. His blood pressure is155/95 mm Hg. Laboratory studies show:
After these findings you began therapy with levothyroxine. Now, 6 months later,
the patient returns for further follow-up and reports feeling much more energeti
c. Provided that he has responded appropriately to the medication and that his T
SH normalizes, what effect would you expect to see on his blood pressure and lip
id profile?

A.
Decreased blood pressure, decreased LDL, and decreased triglycerides
B.
Decreased blood pressure, increased LDL, and increased triglycerides
C.
Increased blood pressure, decreased LDL, and decreased triglycerides
D.

Increased blood pressure, increased LDL, and increased triglycerides


E.
No change in blood pressure or lipid profile

The correct answer is A. Hypothyroidism has important effects on the cardiovascu


lar system. Both blood pressure and lipid profile are adversely affected. Parado
xically, low levels of circulating thyroxine will actually elevate blood pressur
e. The process is mediated through an increase in peripheral vascular resistance
. The effect is most pronounced in those with established hypertension. Hypothyr
oidism also decreases heart rate and myocardial contractility, lowering overall
cardiac output. However, the increase in peripheral vascular resistance is more
prominent, resulting in an overall increase in mean arterial pressure. A screeni
ng TSH to rule out hypothyroidism is a routine part of the workup for secondary
causes of hypertension. Hypothyroidism also decreases lipid clearance from the b
lood by the liver, resulting in elevated serum total, LDL, and triglyceride frac
tions. In a prospective study, 4.2% of those with hyperlipidemia were found to h
ave hypothyroidism. Only those patients with a serum TSH concentration above 10
mU/L had a significant reduction in the serum cholesterol concentration during t
hyroid hormone replacement. The effect of thyroid hormone replacement in the set
ting of subclinical hypothyroidism is usually small and varies considerably. Bec
ause of these two effects, both blood pressure and lipid profiles should improve
during thyroid hormone replacement. Choices B, C, D, and E all describe cardiov
ascular changes that would be unexpected.
A 64-year-old man comes to the clinic for follow-up. He has a 9-year history o
f type II diabetes, an 8-year history of hypertension, and a 6-year history of h
ypercholesterolemia. He reports no specific complaints since his last visit 1 ye
ar ago. He now exercises regularly, playing singles tennis three times per week
during the summer and swimming three or four times per week in winter. He tries
to maintain a low-salt diet and has been taking his medicines regularly. Current
ly he takes metformin 1,000 mg BID, glyburide 20 mg QD, atorvastatin 10 mg QD, a
nd enalapril 10 mg BID. He reports checking his finger-sticks sporadically and n
otes blood glucose readings of 90 to 140 mg/dL. On his last visit his hemoglobin
A1c was 6.8%. Physical examination reveals a well-developed elderly man in no d
istress with a pulse of 68/min and blood pressure of 118/76 mm Hg. The remainder
of his physical examination is unremarkable. Laboratory studies show:
Which of the following is the most appropriate intervention at this time for the
patients diabetes?

A.
Check full laboratory studies, including renal function, in 1 year
B.

Initiate low-dose evening NPH insulin


C.
Recheck hemoglobin A1c in 3 months
D.
Schedule renal ultrasound
E.
Start glipizide 5 mg QD

The correct answer is B. This case highlights the need to understand the appropr
iate management of diabetes mellitus based on results of hemoglobin A1c testing.
Both the American Diabetes Association and the American College of Endocrinolog
y recommend improved glycemic control when the hemoglobin A1c is greater than or
equal to 7%. Inasmuch as this patient is near maximal dose of insulin secretago
gue/sulfonylurea (glyburide) as well as insulin sensitizer (metformin), the next
step would be the addition of a thiazolidinediones, an alpha-glucosidase inhibi
tor or the addition of insulin.
Action needs to be taken with this patient now, so waiting for labs in either 3
months (choice C) or 1 year (choice A) is incorrect. It is true that hemoglobin
A1c values should be followed in 3 months. Yet, given that he has maximized life
style changes and also maximized his current drug regimen, this patient should b
e started on an additional agent before rechecking his long-term glucose control
.
This patient most likely has already developed some degree of diabetic nephropat
hy, given his mild renal insufficiency and proteinuria. However, a renal ultraso
und (choice D) will shed no new light on his current clinical situation. It woul
d be more important to quantify the degree of proteinuria with a 24-hour urine p
rotein or albumin collection.
Adding an additional insulin secretagogue/sulfonylurea (choice E) will not impro
ve his blood glucose control because he is already on the maximal dose of a simi
lar agent.
A 31-year-old diabetic man comes into the emergency department requesting a pr
escription and social services. He states that he recently lost his job and became
homeless. As a result he hasnt taken his insulin in 2 days. He denies any nausea
, vomiting, or abdominal pain. You decide to check his bedside glucose and ask h
im to give you a urine specimen to check for ketones. On his way back from the b
athroom he starts vomiting. The nurse notifies you that his blood glucose is so
high that the bedside machine reads high and does not give an actual value. Bloods
are drawn and sent off to the laboratory, which eventually report a glucose lev
el of 855 mg/dL. His serum bicarbonate is 8 mEq/L and you calculate his anion ga
p to be 34. Which of the following should be included in the initial treatment?

A.
Glucagon by intravenous drip
B.
Insulin by intravenous drip
C.
Intravenous bicarbonate until normalized
D.
Judicious use of intravenous fluids to avoid volume overload
E.
Supplemental phosphorus in the form of potassium phosphate

The correct answer is B. This patient presents with diabetic ketoacidosis (DKA).
It is generally accepted that the ideal way to administer insulin is by continu
ous infusion of small doses of regular insulin through an infusion pump. This ap
proach appears to be more physiologic in producing a more linear fall in serum g
lucose and ketones. It is also associated with fewer complications of hypoglycem
ia, hypokalemia, and hypophosphatemia.
Glucagon (choice A) may be used in cases of hypoglycemia to raise the blood gluc
ose.
Acidotic patients routinely recover from DKA without alkali therapy (choice C).
No studies to date have shown a benefit of using bicarbonate in the treatment of
DKA.
Rapid fluid administration is the single most important initial step in the trea
tment of DKA (choice D). The average adult patient has a water deficit of 100 mL
/kg. Fluid restores intravascular volume and normal tonicity, and lowers serum g
lucose and ketones.
There is no established role for initiating IV potassium phosphate in the initia
l treatment of DKA (choice E). Significant hypophosphatemia tends to develop man
y hours into therapy after the patient is admitted.
A 58-year-old man with poorly controlled emphysema comes to your office compla
ining of purplish striae in the abdomen, as well as a round appearance to his fa
ce, which he has noticed for the last 2 to 3 months. The patient has smoked two
packs of cigarettes per day for the last 40 years and has recently noticed a cha
nge in the pattern of his cough, with grayish-brown sputum production. Other tha
n his COPD he has suffered from no other medical problem. His blood pressure is
170/95 mm Hg in both arms, pulse is 100/min, respirations are 22/min, and temper

ature is 37.2 C (99 F). He has a rounded face, an excess deposit of fat over the
dorsal part of the base of the neck, increased abdominal fat around the waist,
and multiple purplish striae over the abdomen. After you review this patients cli
nical presentation and send blood for routine laboratory studies, which of the f
ollowing is the most appropriate next diagnostic test?

A.
Overnight dexamethasone suppression test
B.
High-dose dexamethasone suppression test
C.
Electrocardiogram
D.
Chest radiograph
E.
CT scan of the abdomen

The correct answer is D. This patient is presenting with symptoms compatible wit
h Cushings disease, a change in the pattern of his cough due to COPD, and new s
putum productionall pointing toward a lung malignancy. He is a heavy smoker with
an 80-pack-year history, which puts him at great risk of lung cancer. We know th
at one of the common paraneoplastic syndromes associated with small-cell lung ca
ncer is production of ectopic ACTH by the tumor. So our first priority in this p
atient is to determine whether he has a lung mass, and the first stepwhich is qui
ck, inexpensive, and of great diagnostic valuewould be to perform a chest x-ray a
nd see if a mass is visible.
Performing an overnight dexamethasone suppression test (choice A) would be the f
irst step in proving the diagnosis of Cushings syndrome; we are suspecting the
cause of this patients symptoms to be secondary to an ectopic production of ACTH
by a small-cell CA of the lung.
Performing a high-dose dexamethasone suppression test (choice B) is typically do
ne once Cushings syndrome is diagnosed. It can distinguish Cushings disease (p
itiutary adenoma) from adrenal tumors or ectopic ACTH production. Plasma ACTH ca
n differentiate primary adrenal neoplasm from ectopic ACTH production.
Although an ECG (choice C) would be part of a routine workup, this test would no
t be of diagnostic value in this patient.
Performing a CT of the abdomen (choice E) is not appropriate as an initial study
in this patient. An abdominal CT scan would be done if lung cancer is suspected

by chest x-ray to look for possible metastasis to the adrenal or liver, which w
ould reinforce our suspicion of a small-cell carcinoma of the lung which causes
early metastasis to adrenals, liver, bone or brain.
A 32-year-old man has a gradual onset of headaches, fatigue, and myalgia. He h
as been previously healthy and is not taking any medications. There has been no
vision change or syncope. Examination shows an enlarged tongue, spaced teeth and
a doughy skin appearance. Laboratory studies show elevated insulin like growth
factor (IGF-I) and prolactin levels. His serum glucose is 150 mg/dL. A glucose t
olerance test is abnormal. Which of the following is the most likely cause of th
ese findings?

A.
Diabetes mellitus
B.
Insulinoma
C.
Pituitary adenoma
D.
Pheochromocytoma
E.
Prolactinoma

The correct answer is C. Growth hormone excess in adults leads to acromegaly. La


boratory findings include an abnormal glucose test whereby the growth hormone co
ncentration is elevated 1 hour after oral administration of 100 grams of glucose
. Mild hyperprolactinemia can be noted. The most frequent cause for the conditio
n is a pituitary adenoma. An hour after glucose administration, growth hormone s
hould normally be suppressed to a value less that 1 mcg/L. IGF-I concentrations
are high because of a high level of growth hormone.
Glucose tolerance testing is a useful screening test for diabetes mellitus and g
lucose intolerance is often seen in acromegaly. The patient may have diabetes bu
t further testing will be needed (choice A).
An insulinoma (choice B) would be characterized by hypoglycemia, decreased gluco
se levels after a glucose challenge and elevated levels of insulin.
Pheochromocytoma (choice D) is characterized by paroxysms of hypertension. Evalu
ation will involve 24-hour collection of urine metanephrines, and abdominal scan

ning to evaluate for an adrenal mass.


Prolactinoma (choice E) will cause an elevated level of serum prolactin. Acromeg
aly can present with mildly elevated prolactin levels as well and scanning of th
e head should be undertaken to define the presumptive pituitary adenoma since ac
romegaly is most often caused by a pituitary adenoma.
A 17-year-old man comes to your office for evaluation of delayed puberty, acco
mpanied by his mother. He has been healthy all his life and grew normally until
his mid-teens, when he failed to develop facial hair or increased muscle mass. H
is voice has never deepened, and he does not have penile erections. He stands 19
0 cm (75 in) tall, weighs 73 kg (160 lb), and has an arm span of 198 cm (78 in).
His parents are both much shorter than he. His facial hair is very sparse and f
ine, limited in distribution to the upper lip and chin. He has only scant amount
s of chest, axillary, and pubic hair. His penis is 1 cm in length, and his testi
cles measure 5 cc each. On further questioning, his mother notes that the patien
t has always had a very limited palate, preferring only bland foods ever since e
arly childhood. Laboratory studies show:
Which of the following is the most likely cause for this patients apparent lack
of pubertal development?

A.
Deficiency of hypothalamic gonadotropin-releasing hormone
B.
Lifelong nutritional deficit
C.
Primary failure of testicular development
D.
Pituitary macroadenoma
E.
Sex chromosomal abnormality

The correct answer is A. This young man suffers from Kallmanns syndrome, or con
genital gonadotropin-releasing hormone deficiency. It is the most common cause o
f idiopathic secondary hypogonadism. This is a defect of development of midline
structures, including the hypothalamus. The result is a lack of GnRH secretion,

causing an absence of LH and FSH secretion. Such patients will have minimal andr
ogen production and so will fail to progress through normal puberty. A common ma
nifestation of Kallmanns syndrome is anosmia, or lack of olfactory sense second
ary to developmental abnormality of the olfactory tracts (another midline struct
ure). Patients are also typically eunuchoid in body habitus and are tall with lo
ng arms. This arises from a lack of the normal androgen-mediated closure of the
epiphyseal plates, resulting in continued long bone growth.
Lifelong nutritional deficit (choice B) appears unlikely in this otherwise healt
hy, tall young man. The overall constellation of physical findings and overt hor
monal deficiencies point to a central endocrine gland dysfunction as the cause.
Primary failure of testicular development (choice C) is incorrect based on the r
esults of laboratory studies. With testicular failure and a normal hypothalamic/
pituitary axis, one would expect LH and FSH levels to be extremely high owing to
absence of feedback inhibition.
Pituitary macroadenoma (choice D) is incorrect on the basis of normal levels of
prolactin and TSH. Although a large nonsecreting pituitary tumor can suppress th
e secretion of LH and FSH to cause secondary hypogonadism, it would be distinctl
y unlikely for it to do so and not suppress prolactin and TSH as well. A prolact
inoma could also suppress other pituitary hormones, but then one would also expe
ct to see elevated prolactin levels and a suppressed TSH.
Sex chromosomal abnormality (choice E) namely Klinefelter syndrome, can present
in a manner similar to this. Small testes, lack of pubertal development, and tal
l stature are all characteristic of an XXY genotype. In Klinefelters, lack of sec
ondary sex characteristics stems from primary testicular failure to produce test
osterone. FSH and LH should be elevated as in other forms of primary testicular
failure. Therefore, Klinefelters can be excluded by the patients anosmia and low l
evels of FSH and LH, indicating a central hormonal defect.
A 32-year-old woman comes to the office for her periodic health maintenance ex
amination. Before this visit her records indicate that she was in excellent heal
th. Lately, however, she has noticed that she has been gaining weight (at least
20 pounds) despite good diet control together with her exercise regimen. She als
o states that her skin had become broken out, like when she was in high school. Th
ere also have been frequent headaches that she blames on needing new glasses. Sh
e does not take any medications and does not drink or smoke. She goes to the gym
5 days a week, working out with a trainer twice a week and going on the treadmi
ll or stair climber the other 3 days. Her temperature is 37.0 C (98.6 F), blood
pressure is 166/92 mm Hg, pulse is 73/min, and respirations are 18/min. Physical
examination shows severe acne on her face and upper back and no palpable cervic
al adenopathy. Heart and lung examinations are normal. There are no palpable abd
ominal masses but centripetal obesity and multiple violaceous striae on her tors
o are appreciated. She has bruises on her legs and arms. Laboratory studies show
:
A CT scan of the abdomen and pelvis shows a 5-cm right adrenal mass. Which of th
e following is the most likely source of this patients problem?

A.
Decreased secretion of glucocorticoids
B.
Increased secretion of adrenocorticotropin hormone

C.
Increased secretion of aldosterone
D.
Increased secretion of glucocorticoids
E.
Increased secretion of norepinephrine and epinephrine

The correct answer is D. This patients symptom complex is diagnostic of Cushings


syndrome, which results from excessive levels of circulating glucocorticoids no
rmally produced by the zona fasciculata and zona reticularis of the adrenal cort
ex. The cause of Cushings syndrome is variable and may be the result of pituita
ry hypersecretion of adrenal corticotropin hormone (ACTH), adrenal adenomas, and
adrenal carcinomas. Other sources of excess ACTH or corticotropin-releasing hor
mone (CRH) include metastatic tumors, such as pulmonary oat cell, carcinoid, epi
thelial carcinoma of the thymus, islet cell tumor of the pancreas, and thyroid c
ancer. The most common cause of Cushins syndrome, however, is the exogenous adm
inistration of corticosteriods. Diagnosis Cushings syndrome is established with
a 24-hour urine free cortisol level and dexamethasone suppression test. Once th
e syndrome has been verified, plasma ACTH and cortisol are measured. Plasma ACTH
is usually undetectable in patients with adrenal tumors and is elevated in pati
ents with an ACTH-secreting adenoma or the pituitary or an ectopic ACTH or CRH-s
ecreting neoplasm. This patient has an adrenal mass; the cortisol will be increa
sed and the ACTH decreased.
Glucocorticoids are essential for life. They exert their effects on a wide spect
rum of cellular metabolism, immune-mediated inflammation, and numerous interacti
ons with other hormones. This patient is manifesting the result of increased glu
cocorticoids, not decreased levels (choice A).
Although it is possible that the adrenal mass is secreting ACTH, it is unlikely
(choice B). In this patient with an adrenal mass, it is more likely that the tum
or is a functioning one and is secreting the glucocorticoids.
Aldosterone is increased in patients with Conns syndrome. The hallmark of this
entity is hypertension in the presence of hypokalemia (choice C).
Pheochromocytomas secrete catecholamines (dopamine, epinephrine, and norepinephr
ine) and there are elevated levels of urinary catecholamines in these patients.
Patients suffering from a pheochromocytoma frequently present with hypertension,
headache, and diaphoresis (choice E).
A previously healthy 32-year-old woman from a small rural town comes to your o
ffice complaining of extreme anxiety and palpitations on several occasions durin
g the past week. She denies having had episodes like these before, and she also
notes feelings of excessive warmth. Her past history is remarkable only for a no
rmal pregnancy six years ago, and she takes no medications. Her pulse is 110/min
and blood pressure is 145/80 mm Hg. Her skin is moist, and she has a rim of scl
era visible above the iris when looking straight ahead. She has a non-palpable t

hyroid and a 2/6 early systolic murmur on cardiac examination. There is a fine t
remor in her hands and she has brisk, hyperactive reflexes bilaterally. Serum th
yroid-stimulating hormone is 0.2 mcU/mL. Her child, who has also been subsequent
ly healthy, accompanies his mother to the clinic today. During the visit, you no
tice that the child is extremely active and on examination is himself tremulous
with a wide-eyed stare. As the towns family physician, you have seen several si
milar cases within the past month. Which of the following is the most likely exp
lanation for these findings?

A.
Accidental ingestion of thyroid hormone
B.
Infectious thyroiditis
C.
Over-supplementation with dietary iodine
D.
Panic attacks
E.
Self-administration of thyroid hormone

The correct answer is A. This patient and her son are clearly suffering from thy
rotoxicosis, as evidenced by the findings on physical examination and blood test
s. Their symptoms are the result of ingestion of thyroid hormone-containing grou
nd beef. Two community outbreaks of thyrotoxicosis have been reported to result
from the accidental ingestion of thyroid tissue from cattle. The tissue was remo
ved along with neck muscle and ground up to prepare hamburger. These outbreaks w
ere labeled "hamburger hyperthyroidism". Affected patients consumed large amount
s of thyroid-hormone-containing ground beef and subsequently became hyperthyroid
until their pantry stocks were depleted. Several weeks thereafter, they became
hypothyroid because of the slow recovery time of the suppressed thyroid.
Infectious thyroiditis (choice B) would seem possible on the basis of this commu
nity "outbreak" and a similar syndrome among family members. However, no virus o
r bacterium has yet been described which infects only the thyroid gland in multi
ple different patients. Subacute (De Quervains) thyroiditis presents in a manner
similar to the case above, except that it usually follows an upper respiratory i
llness. Viral infection and destruction of thyroid follicles result in the relea
se of pre-formed thyroid hormone into the blood, causing thyrotoxicosis. Once th
e infection has resolved, hypothyroidism ensues for a brief period while the gla

nd recovers.
Over-supplementation with dietary iodine (choice C) is possible iatrogenically f
ollowing the administration of iodinated contrast media and amiodarone, an iodin
e-containing medication. However, iodine overdose leads to hypothyroidism, not t
hyrotoxicosis as in this case. Large amounts of iodine suppress the release of t
hyroxine from the thyroid gland, but the effect is transient.
Panic attacks (choice D) are a diagnosis of exclusion once other organic causes
of palpitations and anxiety have been ruled out. In this patients case, she is
clearly hyperthyroid on physical exam, and a suppressed TSH confirms the diagnos
is.
Choice E is incorrect since there is no evidence of self-administration of thyro
id hormone and it is unclear why the patient would give the hormone to her son.
In the entity Munchausen syndrome by proxy, caregivers create illness in their w
ards in order to attract attention and sympathy. In this case, the patient does
not come to you seeking attention through her sons illness, and there is no pri
or history of similar behavior. Further, it does not explain how a community out
break of similar illness could occur.
A 52-year-old patient of yours was started on an additional medication to trea
t dyslipidemia 1 week ago. Since the addition of the last medication, he reports
severe flushing and pruritus, to the point that he does not think he can contin
ue to take the medication. Before starting this new medication, he was on a stat
in for the last year and has had an appropriate diet and exercise plan. At his l
ast visit, however, a fasting lipid panel revealed that the LDL-cholesterol was
still elevated above goal, and that the HDL-cholesterol was low. Given that the
patient has two brothers who have died of coronary heart disease while in their
fifties and that he has a history of hypertension, an aggressive two-medication
treatment strategy for his dyslipidemia was started. Today, vital signs and phys
ical examination are unremarkable. Which of the following is the most appropriat
e management at this time?

A.
Change to a short-acting form of the new medication, recheck in 2 weeks
B.
Check stat creatinine kinase levels and liver function tests, stop treatment
C.
Discontinue the statin, continue with the new medication, re-evaluate in 6 month
s
D.
Prescribe aspirin or antihistamine, continue current treatment
E.
Stop treatment immediately, evaluate for dangerous drug interactions

The correct answer is D. Facial flushing and pruritus are common side effects of
nicotinic acid. Nicotinic acid often is used for treatment of dyslipidemia, as
it can modestly decrease LDL-cholesterol while increasing HDL-cholesterol. Flush
ing, pruritus, and gastrointestinal side effects commonly cause patients to disc
ontinue treatment. Slowly titrating the dose, taking the medication with meals,
and taking an aspirin or antihistamine at least 30 minutes before the medication
can minimize these side effects. Additionally, long-acting formulations, as opp
osed to short-acting forms (choice A), are associated with less flushing.
Liver function tests need to be checked regularly with statins and with nicotini
c acid derivatives. Rarely, statins can cause myopathy, symptoms of which would
warrant checking a creatinine kinase level. Neither creatinine kinase levels nor
liver function tests (choice B), however, are likely to explain this patients sy
mptoms.
There is no reason to discontinue the statin (choice C) or to stop treatment imm
ediately (choice E). These side effects are unlikely to be caused by the statin
or by a drug interaction. Further, whereas nicotinic acid is a good medication a
s an adjunct, it is not as effective as a statin for treating LDL-cholesterol. T
he statin is likely needed and, in any case, is not the cause of this patients co
mplaints.
A 58-year-old woman is found on a routine medical checkup to have a serum calc
ium level of 11.8 mg/dL where the upper limit of normal is 9.5. Repeated determi
nations confirm values between 10.9 and 12.2 mg/dL, and she is found to have ele
vated concentrations of parathyroid hormone. She is asymptomatic, has no pertine
nt family history, and has no evidence of renal stones or bone disease. She is o
ffered the option of elective parathyroidectomy, but she declines and elects to
have close medical follow-up. While doing so, which of the following is a possib
le long-term treatment?

A.
Estrogen replacement
B.
Long-term calcitonin
C.
Low calcium intake
D.
Low-dose thiazides
E.

Vitamin D analogues

The correct answer is A. The underlying pathology is most likely a single parath
yroid adenoma, for which surgical removal is the only cure. However, in postmeno
pausal women estrogen replacement can attenuate bone resorption due to primary h
yperparathyroidism. The risks and benefits of using estrogens must be thoroughly
discussed with the patient.
Calcitonin (choice B) is useful when dealing acutely with extremely high levels
of serum calcium, but long-term therapy is not possible. Resistance to its hypoc
alcemic effects develops within 1 or 2 days.
Low calcium intake (choice C) can result in increased hormone production, and it
can accentuate bone disease. Calcium intake for this woman should be modest, bu
t not low.
Thiazides (choice D) are contraindicated, as they can worsen hypercalcemia (when
diuretics are used to control excessive hypercalcemia, furosemide is recommende
d).
Vitamin D analogues (choice E) are used in patients with hyperparathyroidism sec
ondary to renal failure.
A 28-year-old woman returns to the clinic after being diagnosed with hypothyro
idism 1 week ago. She states that despite taking 100 g of levothyroxine daily, sh
e has had little improvement in her symptoms. She reports continued fatigue and
depressed mood. Vital signs are within normal limits. Physical examination revea
ls a mildly obese woman with coarse hair and skin, delayed relaxation of reflexe
s, and periorbital edema. Her physical examination, although abnormal, is essent
ially the same as when you saw her last week. A repeat thyroid-stimulating hormo
ne (TSH) level is elevated, at a level similar to that taken before starting her
treatment. Which of the following is the most appropriate management?

A.
Check free T4 level, adjust medication appropriately
B.
Decrease levothyroxine dose, recheck TSH in 1 week
C.
Increase in levothyroxine, recheck TSH in 1 week
D.
No change, recheck TSH in 2-4 weeks
E.

Order T3 level, start T3 supplementation

The correct answer is D. It takes approximately 6 weeks before a steady state of


synthetic T4 replacement therapy is achieved. As such, after initiation of levo
thyroxine, the patient should not be reevaluated before 3-6 weeks of treatment.
Three to six weeks (depending on the patients symptoms) after starting therapy, T
SH and T4 should be measured and the dose adjusted accordingly.
Free T4 (choice A) does not need to be monitored at this time. Once the patient
has reached steady state, TSH levels can be used for monitoring.
There is no indication at this time to either decrease (choice B) or increase (c
hoice C) the dose of levothyroxine. Additionally, after adjusting the dose, one
needs to wait 3-6 weeks before checking a TSH and T4.
T3 supplementation (choice E) never should be used to treat hypothyroid disease,
and is best used in specific situations after consultation with an endocrinolog
ist. Replacement T3 is used in certain situations, including refractory depressi
on and thyroid carcinoma.
A 56-year-old woman arrives at the office with a question about postmenopausal
hormone replacement therapy. She has been on estrogen-progestin therapy after h
er menopausal symptoms because of the purported health benefits of replacement t
herapy. She is now concerned with new data about the risks of combined estrogenprogestin therapy, particularly the long term effects of continued use. She has
no chronic medical conditions and has never had any surgery. After 5 years, whic
h of the following conditions is the patient at greatest risk for developing if
continued on combined estrogen-progestin therapy?

A.
Cerebrovascular accident
B.
Colorectal cancer
C.
Death
D.
Vulvovaginal atrophy
E.
Hip fracture

The correct answer is A. The Womens Health Initiative, which began in 1991, was d
etermined to examine the effects of hormone replacement therapy (HRT) in women,
one of the research arms involving postmenopausal women and the incidence of adv
erse events in patients on HRT. The study was anticipated to be completed in 200
5 but was stopped in 2002 because of interim data analysis. This analysis showed
a statistically significant increase in cerebrovascular events (41% increase co
mpared to placebo), cardiovascular disease (i.e., myocardial infarction-29% incr
ease compared to placebo), breast cancer (26% increase compared to placebo), and
deep venous thrombosis and pulmonary embolus.
Colorectal cancer (choice B) incidence decreased 37% during the interim analysis
compared to placebo.
There was no statistically significant difference in death (choice C) outcomes i
n patients on estrogen-progestin hormone replacement compared with placebo.
HRT reduces the vulvovaginal atrophy (choice D)associated with menopause.
Hip fracture (choice E) incidence was reduced by 34% compared to placebo.
A 43-year-old man with a history of diabetes mellitus type 1 comes to the emer
gency department complaining of episodes of palpitations, agitation, and diffuse
sweating increasing over the last 4 months. His medications include lisinopril
and multiple daily injections of NPH and regular insulin, and his dosages have b
een stable for many years. On physical examination, he is an anxious-appearing,
thin man with a blood pressure of 155/95 mm Hg and a pulse of 105/min. He has a
wide-eyed stare and lid lag on downward gaze. Cardiac examination reveals tachyc
ardia, an irregularly irregular rhythm, and a 2/6 early systolic murmur. He has
3+ deep tendon reflexes throughout. His fasting glucose level is 89 mg/dL and hi
s TSH level is undetectable. Which of the following is the most important therap
y for preventing short-term mortality in this patient?

A.
Immediate suppression of thyroid hormone production
B.
Nonselective beta-blockade
C.
Prevention of hypoglycemic episodes
D.
Tight control of blood pressure
E.

Tight control of hyperglycemia

The correct answer is B. This patient has developed hyperthyroidism, most likely
of an autoimmune nature. Type 1 diabetic patients are at increased risk for dev
eloping autoimmune thyroid disease, presumably because of the underlying immune
dysregulation that caused their pancreatic disease. It can present as Hashimoto
thyroiditis, in which the patient is initially hyperthyroid and later becomes hy
pothyroid as the gland is destroyed. They also may develop Graves disease in whi
ch the thyroid is stimulated by an autoantibody. In the initial treatment of thy
rotoxicosis, the most important step is to prevent sudden cardiac arrhythmia. Ex
cessive levels of thyroid hormones increase the hearts sensitivity to circulating
catecholamines. This patient already has atrial fibrillation and tachycardia, i
ndicating a myocardium at risk for a more serious arrhythmia. His initial regime
n should include a nonselective beta-blocker such as propranolol, and he should
continue on it until he has returned to a euthyroid state. These agents block th
e effects of epinephrine and norepinephrine to decrease ventricular automaticity
, slow conduction through the AV node, and lower the rate of the SA node. All of
these effects dramatically reduce the risk for unstable arrhythmias.
Although definitive treatment of this patient will require suppression of excess
ive thyroid hormone production (choice A) either through surgical or pharmacolog
ic means, thyroid function takes weeks to months to decline to low levels and be
ta-blockade cannot be delayed.
This patient has had symptoms suggestive of hypoglycemic episodes (choice C), ho
wever his overall clinical picture indicates hyperthyroidism as the cause. Type
1 diabetic patients are able to maintain a stable dose of insulin over time unle
ss they develop other comorbidities, experience dramatic changes in weight, or t
ake additional medications. He therefore is unlikely to be having hypoglycemic e
pisodes. In fact, his hyperthyroidism is likely to elevate his blood glucose lev
els.
Tight control of blood pressure (choice D) is critical for the prevention of cor
onary artery disease, stroke, and renal insufficiency, especially in diabetics.
Lowering blood pressure through beta-blockade is an important consideration; how
ever, the short-term mortality benefit from these agents in thyrotoxicosis comes
from their ability to protect the myocardium from the arrhythmogenic effects of
catecholamines.
Tight control of hyperglycemia (choice E) has been shown in randomized controlle
d trials to decrease the incidence of microvascular complications (neuropathy, n
ephropathy, and retinopathy) in patients with type 1 diabetes. This benefit has
not been extended yet to a decrease in mortality, and in any case, the patients i
mmediate risk is from his uncontrolled thyrotoxicosis.
A 30-year-old man has chest pain and shortness of breath for the past 4 hours.
He denies prior episodes. He has no medical issues, takes no medications and ha
s no allergies. He reports a strong family history of coronary artery disease. E
valuation reveals positive cardiac enzymes, consistent with a non-Q wave myocard
ial infarction. He is admitted for management. Examination shows nodular swellin
gs on his Achilles tendon. His serum cholesterol level is 400 mg/dL. Which of th
e following protein defects is the most likely cause of this patients condition?

A.
Apoprotein CII
B.
Apoprotein E
C.
LDL receptor
D.
Lipoprotein (a)
E.
Lipoprotein lipase

The correct answer is C. Familial hypercholesterolemia is an autosomal dominant


defect in the LDL receptor. As a result, tissues are unable to take up cholester
ol rich LDL lipoprotein particles from the plasma. The high LDL levels lead to e
nhanced infiltration of cholesterol into the arterial wall causing enhanced athe
rosclerosis.
Apoprotein CII (choice A) deficiency leads to familial apoprotein CII deficiency
. This causes elevation in chylomicrons and VLDL.
Apoprotein E (choice B) defect causes familial type 3 hyperlipoproteinemia and a
n increase in chylomicron remnants. Treatment is with gemfibrozil.
Lipoprotein (a) is a particle that has been suggested to increase the risk of co
ronary artery disease when present in large amounts. It is nearly identical in s
tructure to LDL particles. The particle may slow the breakdown of blood clots (c
hoice D).
Familial lipoprotein lipase deficiency (type 1 hyperlipoproteinemia) is a rare a
utosomal recessive genetic disorder that results from a deficiency of lipoprotei
n lipase. The result is massive chylomicronemia (choice E).
A 25-year-old white man is brought to the emergency department after collapsin
g while playing tennis in the sun. The paramedics who picked him up recorded a b
lood pressure of 65/40 mm Hg at the site, and they started an infusion of Ringers
lactate before the ambulance ride. The patient denies any history of trauma, an
d the absence of trauma was confirmed by bystanders. Initial examination reveals
a blood pressure of 90/60 mm Hg supine, with a pulse of 100/min, but when they
sit him up, the blood pressure becomes 65/40 mm Hg and the pulse increases to 14
0/min. The rest of the physical examination is normal and he looks sun-tanned an
d fit. On questioning, however, he says he has been feeling weak for approximate
ly a year, and he gets dizzy when getting up from bed in the morning. When asked
about sun exposure, he denies much of it before today, expressing puzzlement as

to why he looks so tan, a feature that many of his friends had remarked about.
On further examination, it is ascertained that he also has hyperpigmentation in
the palmar creases and buccal mucosa. Laboratory studies show a serum sodium of
118 mEq/L, serum potassium of 6.5 mEq/L, bicarbonate of 15 mEq/L, and blood gluc
ose of 59 mg/dL. His leukocyte count is 14,000/mm3 with lymphocytosis and eosino
philia. Which of the following is the most likely diagnosis?

A.
Adrenal insufficiency
B.
Cardiogenic shock
C.
Heat stroke
D.
Pituitary insufficiency
E.
Spontaneous rupture of the spleen

The correct answer is A. The hypotension precipitated by sweating and exercising


in the sun, together with the classic electrolyte pattern, point to adrenal ins
ufficiency, a suspicion that is confirmed by the rest of the history and the typ
ical hyperpigmentation.
Cardiogenic shock (choice B) can happen to healthy young men who fall prey to fu
lminating viral myocarditis, but in that case the hypotension is accompanied by
other signs of congestive failure, and the laboratory studies would not have any
of the findings present here.
Heat stroke (choice C) is another good guess for anyone who gets into trouble wh
ile exercising under a hot sun. Missing features for that diagnosis, however, in
clude body temperature (not mentioned anywhere as being abnormal), and the flush
ed, red skin that is very different from the bronzed appearance described here.
Pituitary insufficiency (choice D) could produce secondary adrenal insufficiency
, leading to the same presentation, except for a very important detail: the hype
rpigmentation speaks of an active pituitary trying to stimulate a nonresponding
adrenal cortex.
Shock and bleeding often go together, and the spleen can indeed be fragile, but
spontaneous rupture (choice E) is rare, and when it happens there are usually ab
dominal findings.

A 21-year-old primigravid woman at 22 weeks gestation is brought to the emergen


cy department because of nausea, vomiting, and mental status changes. Her mother
states that her daughters symptoms started earlier in the day and have been grow
ing worse. Her past medical history is significant for diabetes that she has had
since the age of 11. She has never had surgery. She takes insulin, although she
frequently skips doses and days. She has no allergies. Her temperature is 37 C
(98.6 F), pulse is 116/min, blood pressure is 118/74 mm Hg, and respirations are
20/min. The remainder of her physical examination is within normal limits. Arte
rial blood gas analysis reveals a pH of 7.33, pO2 of 90 mm Hg, and pCO2 of 30 mm
Hg. Laboratory studies show a decreased bicarbonate level and a serum glucose o
f 310 mg/dL. Urinalysis shows trace protein and 4+ ketones in the urine. Which o
f the following is the most likely diagnosis?

A.
Antepartum psychosis
B.
Diabetic ketoacidosis
C.
Hyperemesis gravidarum
D.
Hypoglycemic coma
E.
Preeclampsia

The correct answer is B. Diabetic ketoacidosis (DKA) is one of the most serious
complications that diabetic women face during pregnancy. It is a rare complicati
on, affecting only 1% of all of the diabetic pregnancies, but the results can be
catastrophic for the mother and the fetus. Fetal loss is as high as 20% with ma
ternal DKA. DKA can occur due to maternal infection, administration of corticost
eroids, hyperemesis, use of beta-sympathomimetic drugs, or noncompliance with di
et and insulin regimens. This patient has elevated glucose, low bicarbonate, and
an arterial pH of 7.33. She also has 4+ ketones in her urine. All of these find
ings are consistent with a diagnosis of DKA. Treatment of DKA involves the aggre
ssive administration of intravenous fluids (normal saline), insulin, and potassi
um.
Antepartum psychosis (choice A) can occur in some patients, although postpartum
psychosis is a more widely described condition in pregnancy. This patient has al

tered mental status, but this is most likely secondary to her DKA, which is the
most likely diagnosis.
Hyperemesis gravidarum (choice C) is also associated with nausea and vomiting, b
ut it is typically a first-trimester condition that resolves during the second t
rimester. This patient does have nausea and vomiting, but she also has several o
ther findings that point to the diagnosis of DKA.
Hypoglycemic coma (choice D) can affect diabetic patients who take, or are given
, too high a dose of insulin. This patient, however, is neither hypoglycemic nor
in a coma, and therefore does not have this condition.
Preeclampsia (choice E) is diagnosed on the basis of hypertension and proteinuri
a. This patient is normotensive and has only minimal protein in her urine. There
fore, she does not qualify for the diagnosis of preeclampsia.
A 62-year-old noninsulin-dependent diabetic comes to the clinic for follow-up f
or his diabetes. He has been traveling for 2 years as a salesman and has been se
eing doctors in each city as necessary for insulin and glucometer prescriptions.
He has just landed an office job in the company this fall and wants to make sur
e his diabetes is being watched carefully by a physician. He is otherwise active
and exercises about 30 minutes per day, five times a week. He currently takes m
etformin for diabetes and atenolol for mildly elevated blood pressure. He does n
ot drink, smoke, or use illicit drugs. His only major illness was at age 14 when
he developed multiple pruritic vesicles over his body that resolved in 10 days
with no further sequelae. He received a shot 2 years ago to prevent the pneumonia
. Physical examination is unremarkable; there is no evidence of retinopathy or ne
uropathy. He is concerned about making sure that he stays well because a friend
of his with diabetes was recently admitted to the hospital with something called s
epsis. He wants to get his necessary shots before work starts in 5 days. Which of th
e following vaccinations should be administered at this time?

A.
Influenza
B.
Pertussis
C.
Pneumococcal
D.
Tetanus booster
E.
Varicella

The correct answer is A. Patients in high-risk groups, such as patients who are
immunocompromised (e.g., with HIV or leukemia) as well as those at high risk for
systemic infections should have prophylactic immunizations against pneumococcus
and influenza. In addition, they should be up to date on their other vaccinatio
ns as well. In this particular patient yearly influenza vaccinations should be a
dministered because patients with diabetes mellitus are considered to be in a hi
gh-risk group.
Pertussis (choice B) is not recommended after age 4 to 6. Administration of the
vaccine is usually during infancy, with four doses being received by age 18 mont
hs.
Pneumococcal vaccine (choice C) has already been administered to the patient. A
second dose of vaccine should be given if a patient received a vaccine greater t
han or equal to 5 years previously and were aged 65 at the time of vaccination.
Tetanus booster vaccinations (choice D) should be administered every 10 years an
d may be given within that period if there is a question of susceptibility or if
a person is at high risk (i.e., after an open trauma, stabbing, gunshot wound).
Varicella (choice E) vaccination does not appear to be necessary in this patient
, given a history in his youth of chickenpox; immunity is usually conferred afte
r a single disease episode.
A 56-year-old woman with a long history of hypothyroidism comes for a routine
clinic appointment. She is concerned that her hypothyroidism is getting worse. A
lthough her symptoms had been well controlled previously with supplemental levot
hyroxine, over the last 6 months she has noticed increasing malaise, weight gain
, and temperature intolerance, all symptoms that she suffered from before starti
ng treatment. She reports taking her medications as prescribed and has had no ma
jor illness over the past year. Her only recent medical problems have involved g
etting her LDL-cholesterol under control. This goal has been met, however, thoug
h it required the addition of cholestyramine in addition to the statin she was t
aking already. Which of the following is the most likely underlying cause of thi
s patients symptoms?

A.
Coexisting major depressive disorder mimicking hypothyroid state
B.
Dietary indiscretion resulting in reduced drug absorption and utilization
C.
Drug interaction with cholestyramine resulting in reduced bioavailable levothyro
xine
D.
Natural progression of thyroid disease; patients require more medication over ti
me

E.
Occult malignancy or illness resulting in increased utilization of thyroid hormo
ne

The correct answer is C. Cholestyramine is a bile acid sequestrant that can lowe
r LDL-cholesterol and modestly increase HDL-cholesterol when combined with a sta
tin. In addition to binding bile acids in the gut lumen, cholestyramine also can
bind other medications, resulting in reduced absorption and availability of the
medication.
Major depressive disorders (choice A) can mimic hypothyroidism, though temperatu
re intolerance is uncommon. There is no evidence this patient is suffering from
a major depressive disorder.
Dietary indiscretion (choice B) is unlikely to affect the amount of thyroid horm
one available. Other medications, in particular warfarin, do require adherence t
o a set diet and can be affected by dietary indiscretion.
In general, patients do not require more medication over time (choice D). If a p
atient begins to have worsening symptoms with no change in drug dosage, an alter
native cause needs to be found.
There is no evidence this patient has an occult malignancy or illness (choice E)
, though both situations could result in increased utilization of thyroxine.
A 32-year-old woman is being evaluated for amenorrhea for the past 2 months. S
he has been previously healthy and is not taking any medications. She denies any
recent headaches, family history of amenorrhea, excessive exercise, headache, w
eight gain, weight loss, galactorrhea or anorexia. Her menses began at age 10 an
d have been regular until 2 months ago. Urine human chorionic gonadotropin (hCG)
test is negative. Evaluation for polycystic ovarian syndrome (PCOS) is initiate
d. Further evaluation to determine her estrogen status is initiated. Which of th
e following will help establish this diagnosis?

A.
Absence of hot flashes
B.
Elevated follicle stimulating hormone level
C.
Normal estrone level
D.
Normal serum prolactin level

E.
Onset of menses with short course of progesterone

The correct answer is E. This patients negative beta human chorionic gonadotropin
level rules out pregnancy. Secondary amenorrhea exists when menstruation has be
en present and has ceased. Progesterone administration results in secretory diff
erentiation of an estrogen primed endometrium. In a patient being evaluated for
secondary amenorrhea, the appearance of menses after a short course of progester
one is indicative of an estrogen primed endometrium. This provides evidence of o
varian estrogen secretion.
Hot flashes are common in a low estrogen state such as menopause. Hot flashes sh
ould not be seen in an estrogen-exposed state (choice A). Hot flashes are not se
en in hypothalamic amenorrhea-- a common cause of secondary amenorrhea and a low
estrogen state.
In menopause, the estrogen level may be low. The negative feedback loop is remov
ed and the follicle stimulating hormone (FSH) level may be high. In polycystic o
varian disease, estrogen exerts a positive feedback on FSH secretion resulting i
n an elevated luteinizing hormone to FSH ratio (choice B).
Estrone levels are not indicative of direct ovarian estrogen secretion since it
is derived primarily from the peripheral conversion of androstenedione (choice C
).
A normal plasma prolactin level (choice D) does not indicate a low estrogen stat
e.

Anda mungkin juga menyukai